Midterm Study Guide

अब Quizwiz के साथ अपने होमवर्क और परीक्षाओं को एस करें!

A nurse cares for a client with diabetes mellitus who asks, "Why do I need to administer more than one injection of insulin each day?" How would the nurse respond?

"A single dose of insulin each day would not match your blood insulin levels and your food intake patterns."

A nurse teaches assistive personnel (AP) about how to care for a client with Parkinson disease. Which statement would the nurse include as part of this teaching?

"Allow the client to be as independent as possible with activities."

A nurse prepares to discharge a client who has heart failure. Which questions would the nurse ask to ensure this client's safety prior to discharging home? (Select all that apply.)

"Are your bedroom and bathroom on the first floor?" , "What social support do you have at home?" , "What spiritual beliefs may impact your recovery?"

A nurse teaches a client who has chronic obstructive pulmonary disease. Which statements related to nutrition would the nurse include in this client's teaching? (Select all that apply.)

"Avoid drinking fluids just before and during meals." , "Rest before meals if you have dyspnea." , "Have about six small meals a day.", "Use pursed-lip breathing during meals." , "Choose soft, high-calorie, high-protein foods."

A nurse is providing discharge teaching to a client recovering from a heart transplant. Which statement would the nurse include?

"Avoid large crowds and people who are sick."

A nurse cares for a client who has a heart rate averaging 56 beats/min with no adverse symptoms. Which activity modification would the nurse suggest to avoid further slowing of the heart rate?

"Avoid straining while having a bowel movement."

A nurse is teaching a client with heart failure who has been prescribed enalapril. Which statement would the nurse include in this client's teaching?

"Avoid using salt substitutes."

A nurse teaches a client with heart failure about energy conservation. Which statement would the nurse include in this client's teaching?

"Begin walking 200 feet a day three times a week."

A nurse is assessing a client with peripheral artery disease (PAD). The client states that walking five blocks is possible without pain. What question asked next by the nurse will give the best information?

"Could you walk further than that a few months ago?"

After a hospital's emergency department (ED) has efficiently triaged, treated, and transferred clients from a community disaster to appropriate units, the hospital incident command officer wants to "stand down" from the emergency plan. Which question would the nursing supervisor ask at this time?

"Do all areas of the hospital have the supplies and personnel they need?"

A nurse teaches a patient about self-monitoring of blood glucose levels. Which statement would the nurse include in this client's teaching to prevent bloodborne infections?

"Do not share your monitoring equipment."

A nurse teaches a client who is prescribed digoxin therapy. Which statement would the nurse include in this client's teaching?

"Do not take this medication within 1 hour of taking an antacid."

A nurse teaches a client with diabetes mellitus about foot care. Which statements would the nurse include in this client's teaching? (Select all that apply.)

"Do not walk around barefoot." , "Trim toenails straight across with a nail clipper."

A nurse cares for a client with chronic obstructive pulmonary disease (COPD) who appears thin and disheveled. Which question would the nurse ask first?

"Do you experience shortness of breath with basic activities?"

The nurse is caring for a client who has Alzheimer disease. The client's wife states, "I am having trouble managing his behaviors at home." Which questions would the nurse ask to assess potential causes of the client's behavior problems? (Select all that apply.)

"Does your husband bathe and dress himself independently?" , "Does his behavior become worse around large crowds?" , "Do you have a clock and calendar in the bedroom and kitchen?"

A nurse teaches the spouse of a client who has Alzheimer disease. Which statements should the nurse include in this teaching related to caregiver stress reduction? (Select all that apply.)

"Establish advanced directives early." , "Set aside time each day to be away from the client." , "Use discipline to correct inappropriate behaviors."

After teaching a client who is diagnosed with new-onset epilepsy and prescribed phenytoin, the nurse assesses the client's understanding. Which statement by the client indicates a correct understanding of the teaching?

"Even when my seizures stop, I will continue to take this drug."

A nurse assesses the health history of a client who is prescribed ziconotide for chronic low back pain. Which assessment question would the nurse ask?

"Have you been diagnosed with a mental health problem?"

A male client was admitted with a left-sided stroke this morning. The assistive personnel asks about meeting the client's nutritional needs. Which response by the nurse is appropriate?

"He is NPO until the speech-language pathologist performs a swallowing evaluation."

After teaching the wife of a client with Parkinson's disease, the nurse assesses the wife's understanding. Which statement by the client's wife indicates that she correctly understands the changes associated with this disease?

"He may have trouble chewing, so I will offer bite-sized portions."

A nurse teaches a community health class about water safety. Which statement by a participant indicates that additional teaching is needed?

"I can go swimming all by myself because I am a certified lifeguard."

A nurse cares for an older adult client with heart failure. The client states, "I don't know what to do. I don't want to be a burden to my daughter, but I can't do it alone. Maybe I should die." What is the best response by the nurse?

"I can stay if you would you like to talk more about this."

A client has peripheral arterial disease (PAD). What statement by the client indicates misunderstanding about self-management activities?

"I can use a heating pad on my legs if it's set on low."

The nurse observes a client with late-stage Alzheimer disease eat breakfast. Afterward the client states, "I am hungry and want breakfast." What is the nurse's best response?

"I see you are still hungry. I will get you some toast."

The nurse is preparing to teach a client recently diagnosed with multiple sclerosis about taking glatiramer acetate. Which statement by the client indicates a need for further teaching?

"I should report any flulike symptoms to my primary health care provider."

After providing discharge teaching, a nurse assesses the client's understanding regarding increased risk for metabolic alkalosis. Which statement indicates that the client needs additional teaching?

"I take sodium bicarbonate after every meal to prevent heartburn."

The nurse is teaching a family caregiver about how best to communicate with the client who has been diagnosed with Alzheimer disease. Which statement by the caregiver indicates a need for further teaching?

"I will avoid communicating with the client to prevent agitation."

After teaching a client who has an implantable cardioverter-defibrillator (ICD), a nurse assesses the client's understanding. Which statement by the client indicates correct understanding of the teaching?

"I will avoid sources of strong electromagnetic fields."

After teaching a client who was malnourished and is being discharged, a nurse assesses the client's understanding. Which statement indicates that the client correctly understood teaching to decrease risk for the development of metabolic acidosis?

"I will eat three well-balanced meals and a snack daily."

After teaching a client who is prescribed a long-acting beta2 agonist medication, a nurse assesses the client's understanding. Which statement indicates that the client comprehends the teaching?

"I will take this medication every morning to help prevent an acute attack."

After teaching a client who is prescribed salmeterol, the nurse assesses the client's understanding. Which statement by the client indicates a need for additional teaching?

"I will use the drug when I have an asthma attack."

A nurse cares for a client with chronic obstructive pulmonary disease (COPD). The client states that going out with friends is no longer enjoyable. How would the nurse respond?

"I'd like to hear about thoughts and feelings causing you to limit social activities."

A client is scheduled for a percutaneous endoscopic lumbar discectomy. Which statement by the client indicates a need for further teaching?

"I'll be in the hospital for 2 to 3 days."

"If I am nauseated, I will not take my epilepsy medication."After teaching a client newly diagnosed with epilepsy, the nurse assesses the client's understanding. Which statement by the client indicates a need for additional teaching?

"If I am nauseated, I will not take my epilepsy medication."

A nurse cares for victims during a community-wide disaster drill. One of the victims asks, "Why are the individuals with black tags not receiving any care?" How does the nurse respond?

"In a disaster, extensive resources are not used for one person at the expense of many others."

A client has a brain tumor and is receiving phenytoin (Dilantin). The spouse questions the use of the drug, saying that the client does not have a seizure disorder. What response by the nurse is correct?

"Increased pressure from the tumor can cause seizures."

A client asks what "essential hypertension" is. What response by the registered nurse is best?

"It is hypertension with no specific cause."

A primary health care provider prescribes diazepam to a client who was bitten by a black widow spider. The client asks, "What is this medication for?" How does the nurse respond?

"It will relieve your muscle rigidity and spasms."

An older client with peripheral vascular disease (PVD) is explaining the daily foot care regimen to the family practice clinic nurse. What statement by the client may indicate a barrier to proper foot care?

"My hands shake when I try to do things requiring coordination."

After teaching a client how to prevent altitude-related illnesses, a nurse assesses the client's understanding. Which statement indicates that the client needs additional teaching?

"My partner and I will plan to sleep at a higher elevation to acclimate more quickly."

A nurse assesses a client admitted to the cardiac unit. Which statement by the client alerts the nurse to the possibility of right-sided heart failure?

"My shoes fit really tight lately."

A nurse is teaching a female client about alcohol intake and how it affects hypertension. The client asks if drinking two beers a night is an acceptable intake. What answer by the nurse is best?

"No, women should only have one beer a day as a general rule."

The nurse teaches assistive personnel (AP) about how to care for a client with early-stage Alzheimer disease. Which statement would the nurse include?

"Reorient the client to the day, time, and environment with each contact."

The primary health care provider prescribes donepezil for a client diagnosed with early-stage Alzheimer disease. What teaching about this drug will the nurse provide for the client's family caregiver?

"Report any client dizziness or falls because the drug can cause bradycardia."

A nurse collaborates with assistive personnel (AP) to provide care for a client with congestive heart failure. Which instructions would the nurse provide to the AP when delegating care for this client? (Select all that apply.)

"Reposition the client every 2 hours." , "Accurately record intake and output." , "Use the same scale to weigh the client each morning."

A nurse cares for a client who has a family history of cystic fibrosis. The client asks, "Will my children have cystic fibrosis?" How would the nurse respond?

"Since you have a family history of cystic fibrosis, I would encourage you and your partner to be tested."

After teaching a client with diabetes mellitus to inject insulin, the nurse assesses the client's understanding. Which statement made by the client indicates a need for further teaching?

"The lower abdomen is the best location because it is closest to the pancreas."

A nurse is caring for a client with paraplegia who is scheduled to participate in a rehabilitation program. The client states, "I don't understand the need for rehabilitation; the paralysis will not go away and it will not get better." How would the nurse respond?

"The rehabilitation program will teach you how to maintain the functional ability you have and prevent further disability."

A nurse cares for a client who had a partial laryngectomy 10 days ago. The client states that all food tastes bland. How would the nurse respond?

"This is normal after surgery. What types of food do you like to eat?"

A nurse in a family practice clinic is preparing discharge instructions for a client reporting facial pain that is worse when bending over, tenderness across the cheeks, and postnasal discharge. What instruction will be most helpful?

"Try warm, moist heat packs on your face."

A nurse teaches a client who has severe allergies ways to prevent insect bites. Which statements does the nurse include in this client's teaching? (Select all that apply.)

"Use screens in your windows and doors to prevent flying insects from entering.", "Do not swat at insects or wasps.", "Identify and remove potential nesting sites in your yard.", "Consult an exterminator to control bugs in and around your home."

A nurse is teaching a client how to perform pursed-lip breathing. Which instructions would the nurse include in this teaching? (Select all that apply.)

"Use your abdominal muscles to squeeze air out of your lungs." , "Breath out slowly without puffing your cheeks." , "Exhale at least twice the amount of time it took to breathe in."

A nurse is teaching a wilderness survival class. Which statements would the nurse include about the prevention of hypothermia and frostbite? (Select all that apply.)

"Wear synthetic clothing instead of cotton to keep your skin dry.", "Know your physical limits. Come in out of the cold when limits are reached.", "Change your gloves and socks if they become wet."

A client has been admitted for suspected inhalation anthrax infection. What question by the nurse is most important?

"What is your occupation?"

A client who is hospitalized with burns after losing the family home in a fire becomes angry and screams at a nurse when dinner is served late. How would the nurse respond?

"You seem upset. I have time to talk if you'd like."

A nurse is caring for a client whose spouse died in a recent mass casualty accident. The client says, "I can't believe that my spouse is gone and I am left to raise my children all by myself." How would the nurse respond?

"You sound anxious about being a single parent."

A nurse is teaching a client with diabetes mellitus who asks, "Why is it necessary to maintain my blood glucose levels no lower than about 60 mg/dL (3.3 mmol/L)?" How would the nurse respond?

"Your brain needs a constant supply of glucose because it cannot store it."

After teaching a client who is recovering from a heart transplant to change positions slowly, the client asks, "Why is this important?" How would the nurse respond?

"Your new heart is not connected to the nervous system and is unable to respond to decreases in blood pressure caused by position changes."

A nurse cares for a client who has a family history of diabetes mellitus. The client states, "My father has type 1 diabetes mellitus. Will I develop this disease as well?" How would the nurse respond?

"Your risk of diabetes is higher than the general population, but it may not occur."

A nurse caring for a client removes the client's oxygen as prescribed. The client is now breathing what percentage of oxygen in the room air?

21%

The nurse is teaching assistive personnel (AP) about fluid restriction for a client who has acute kidney injury (AKI). The client's 24-hour urinary output is 120 mL. How much fluid would the client be allowed to have over the next 24 hours?

620 mL

A nurse teaches a client who is being discharged after a jaw wiring for a mandibular fracture. Which statements would the nurse include in this patient's teaching? (Select all that apply.)

A 24-year-old with a traumatic brain injury , A 58-year-old getting radiation therapy , A 66 year old who is a quadriplegic

An emergency department nurse is triaging victims of a multi-casualty event. Which client would receive care first?

A 26-year-old male who has pale, cool, clammy skin.

The nurse assesses clients on a medical-surgical unit. Which clients would the nurse identify as at risk for secondary seizures? (Select all that apply.)

A 26-year-old woman with a left temporal brain tumor , A 38-year-old male client in an alcohol withdrawal program , A 42-year-old football player with a traumatic brain injury

A nurse assesses clients on a cardiac unit. Which client would the nurse identify as being at greatest risk for the development of left-sided heart failure?

A 36-year-old woman with aortic stenosis

A nurse assesses clients on a cardiac unit. Which clients would the nurse identify as at greatest risk for the development of acute pericarditis? (Select all that apply.)

A 36-year-old woman with systemic lupus erythematosus (SLE) , A 42-year-old man recovering from coronary artery bypass graft surgery , An 80-year-old man with a bacterial infection of the respiratory tract

A nurse is triaging clients in the emergency department (ED). Which client would the nurse prioritize to receive care first?

A 45-year-old reporting chest pain and diaphoresis., A 22-year-old with a painful and swollen right wrist.

A nurse is assessing clients on a medical-surgical unit. Which client would the nurse identify as being at greatest risk for atrial fibrillation?

A 50-year-old who is post coronary artery bypass graft surgery.

A pulmonary nurse cares for clients who have chronic obstructive pulmonary disease (COPD). Which client would the nurse assess first?

A 52 year old in a tripod position using accessory muscles to breathe

A nurse assesses clients who are at risk for diabetes mellitus. Which client is at greatest risk?

A 58-year-old American Indian

A nurse is triaging clients in the emergency department. Which client would the nurse classify as "nonurgent?"

A 62-year-old with a simple fracture of the left arm.

A nurse is triaging clients in the emergency department. Which client would be considered "urgent"?

A 75-year-old female with a cough and a temperature of 102° F (38.9° C)

A nurse is teaching a client about possible complications and hazards of home oxygen therapy. About which complications does the nurse plan to teach the client? (Select all that apply.)

Absorptive atelectasis , Combustion , Dried mucous membranes , Toxicity

A client is undergoing hemodialysis. The client's blood pressure at the beginning of the procedure was 136/88 mm Hg, and now it is 110/54 mm Hg. What actions would the nurse perform to maintain blood pressure? (Select all that apply.)

Adjust the rate of extracorporeal blood flow. , Place the patient in the Trendelenburg position. , Administer a 250-mL bolus of normal saline.

A nurse assesses a client after administering the first dose of a nitrate. The client reports a headache. What action would the nurse take?

Administer PRN acetaminophen.

An emergency department nurse cares for a middle-age mountain climber who is confused, ataxic, and exhibits impaired judgement. After administering oxygen, which intervention would the nurse implement next?

Administer dexamethasone.

The nurse is caring for a client who has severe hypoglycemia and is experiencing a seizure. What actions will the nurse take at this time? (Select all that apply.)

Administer glucagon 1 mg subcutaneously. , Be sure the bedside rails are in the up position. , Notify the primary health care provider immediately. , Monitor the client's blood glucose level.

A primary health care provider prescribes a rewarming bath for a client who presents with Grade 3 frostbite. What action would the nurse take prior to starting this treatment?

Administer intravenous morphine.

A middle-age mountain hiker is admitted to the emergency department exhibiting a cough with pink, frothy sputum and cyanosis of lips and nail beds. What priority action would the nurse implement?

Administer oxygen via a nonrebreather mask.

A nurse cares for a client who had a chest tube placed 6 hours ago and refuses to take deep breaths because of the pain. What action would the nurse take?

Administer pain medication and encourage the client to take deep breaths.

A client presents to the emergency department after prolonged exposure to the cold. The client is difficult to arouse and speech is incoherent. What action would the nurse take first?

Administer warmed intravenous fluids to the client.

A nurse cares for older clients who have traumatic brain injury. What does the nurse understand about this population? (Select all that apply.)

Admission can overwhelm the coping mechanisms for older clients. , These clients are more susceptible to systemic and wound infections. , Other medical conditions can complicate treatment for these clients.

A nurse is assessing a client who has suffered a nasal fracture. Which assessment would the nurse perform first?

Airway patency

A 70-kg adult client with chronic kidney disease (CKD) is on a 40-g protein diet. The patient has a reduced glomerular filtration rate and is not undergoing dialysis. Which result would be of most concern to the nurse?

Albumin level of 2.5 g/dL (3.63 mcmol/L)

The nurse is caring for a client with a new diagnosis of chronic kidney disease. Which priority complications would the nurse anticipate? (Select all that apply.)

Anemia, Hypertension, Dysrhythmias, Heart failure

The emergency department (ED) manager is reviewing client charts to determine how well the staff perform when treating clients with community-acquired pneumonia. What outcome demonstrates that goals for this client type have been met?

Antibiotics started before admission.

A client has a deep vein thrombosis (DVT). What comfort measure does the nurse delegate to the assistive personnel (AP)?

Apply a warm moist pack.

A nurse is in charge of a first-aid tent at an all-day outdoor sports event on a hot and humid day. A participant comes to the tent reporting a headache, weakness, and nausea. What actions would the nurse take? (Select all that apply.)

Apply cold packs to neck, arm pits, and groin., Encourage drinking a sports drink., Have the client lie down in a cool place.

A client is receiving oxygen at 4 L per nasal cannula. What comfort measure may the nurse delegate to assistive personnel (AP)?

Apply water-soluble ointment to nares and lips.

What nonpharmacologic comfort measures would the nurse include in the plan of care for a client with severe varicose veins? (Select all that apply.)

Applying elastic compression stockings , Elevating the legs when sitting or lying , Reminding the client to do leg exercises

A client has a tracheostomy tube in place. When the nurse suctions the client, food particles are noted. What action by the nurse is inappropriate?

Applying suction while inserting the catheter

A nurse is demonstrating suctioning a tracheostomy during the annual skills review. What action by the student demonstrates that more teaching is needed?

Applying suction while inserting the catheter

A client had a femoral-popliteal bypass graft with a synthetic graft. What action by the nurse is most important to prevent wound infection?

Appropriate hand hygiene before giving care.

An emergency department charge nurse notes an increase in sick calls and bickering among the staff after a week with multiple trauma incidents. What action would the nurse take?

Arrange for post-incident crisis support.

A nurse cares for a client with arthritis who reports frequent asthma attacks. What action would the nurse take first?

Ask about medications the client is currently taking.

A nurse plans care for a client who has chronic obstructive pulmonary disease and thick, tenacious secretions. Which interventions would the nurse include in this client's plan of care? (Select all that apply.)

Ask the client to drink 2 L of fluids daily. , Add humidity to the prescribed oxygen. , Use a vibrating chest physiotherapy device. , Administer the ordered mucolytic agent.

A nurse performs an admission assessment on a 75-year-old client with multiple chronic diseases. The client's blood pressure is 135/75 mm Hg and oxygen saturation is 94% on 2 L per nasal cannula. The nurse assesses the client's rhythm on the cardiac monitor and observes the reading shown below:

Ask the client what medications he or she takes.

A client with a known abdominal aortic aneurysm reports dizziness and severe abdominal pain. The nurse assesses the client's blood pressure at 82/40 mm Hg. What actions by the nurse are most important? (Select all that apply.)

Assess distal pulses every 10 minutes. , Notify the Rapid Response Team. , Take vital signs every 10 minutes.

While assessing a client on a cardiac unit, a nurse identifies the presence of an S3 gallop. What action would the nurse take next?

Assess for symptoms of left-sided heart failure.

Emergency medical technicians arrive at the emergency department with an unresponsive client who has an oxygen mask in place. Which action would the nurse take first?

Assess that the client is breathing adequately.

A nurse is caring for a client on IV infusion of heparin. What actions does this nurse include in the client's plan of care? (Select all that apply.)

Assess the client for bleeding. , Monitor the daily activated partial thromboplastin time (aPTT) results. , Use an IV pump for the infusion.

A client is admitted with a sudden decline in level of consciousness. What is the nursing action at this time?

Assess the client for hypoglycemia and hypoxia.

A nurse cares for a client who has packing inserted for posterior nasal bleeding. What action would the nurse take first?

Assess the client's airway.

A client with a history of heart failure and hypertension is in the clinic for a follow-up visit. The client is on lisinopril and warfarin. The client reports new-onset cough. What action by the nurse is most appropriate?

Assess the client's lung sounds and oxygenation.

After a craniotomy, the nurse assesses the client and finds dry, sticky mucous membranes, acute confusion, and restlessness. The client has IV fluids running at 75 mL/hr. What action by the nurse would the nurse take first?

Assess the client's serum sodium level.

A nurse plans care for a client with a halo fixator. Which interventions would the nurse include in this client's plan of care? (Select all that apply.)

Assess the pin sites for signs of infection. , Assess the chest and back for skin breakdown.

A client has been diagnosed with a deep vein thrombosis and is to be discharged on warfarin. The client is adamant about refusing the drug because "it's dangerous." What action by the nurse is best?

Assess the reason behind the client's fear.

A nurse cares for a client with an intravenous temporary pacemaker for bradycardia. The nurse observes the presence of a pacing spike but no QRS complex on the client's electrocardiogram. What action would the nurse take next?

Assess vital signs and level of consciousness.

A client has hypertension and high risk factors for cardiovascular disease. The client is overwhelmed with the recommended lifestyle changes. What action by the nurse is best?

Assist in finding one change the client can control.

A nurse cares for a client who is scheduled for a total laryngectomy. What action would the nurse take prior to surgery?

Assist the client to choose a communication method.

A client has been diagnosed with an empyema. What interventions would the nurse anticipate providing to this client? (Select all that apply.)

Assisting with chest tube insertion , Facilitating pleural fluid sampling, Performing frequent respiratory assessment , Providing antipyretics as needed

The nurse working in the emergency department knows that which factors are commonly related to aneurysm formation? (Select all that apply.)

Atherosclerosis, History of hypertension , History of smoking, Hyperlipidemia

A nurse assesses a client who has mitral valve regurgitation. For which cardiac dysrhythmia would the nurse assess?

Atrial fibrillation

The nurse is evaluating a 3-day diet history with a client who has an elevated lipid panel. What meal selection indicates that the client is managing this condition well with diet?

Baked chicken breast, broccoli, tomatoes

Emergency medical services (EMS) brings a large number of clients to the emergency department following a mass casualty incident. The nurse identifies the clients with which injuries with yellow tags? (Select all that apply.)

Bruising and pain in the right lower abdomen, Partial-thickness burns covering both legs, Small pieces of shrapnel embedded in both eyes, Neck injury and numbness of both legs

A client with chronic kidney disease (CKD) has an elevated serum phosphorus level. What drug would the nurse anticipate to be prescribed for this client?

Calcium acetate

A client who had therapeutic hypothermia after a traumatic brain injury is slowly rewarmed to a normal core temperature. For which assessment finding would the nurse monitor during the rewarming process?

Cardiac dysrhythmias

A client admitted to the emergency department following a lightning strike. What is the priority assessment the nurse focuses on?

Cardiopulmonary

The nurse is teaching participants in a family-oriented community center, ways to prevent their older relatives and friends from getting heat-related illnesses. What information does the nurse include? (Select all that apply.)

Check on the older adult daily in hot weather., Take cool baths or showers after outdoor activities., Drink plenty of liquids throughout the day.

A nurse administers medications to a client who has asthma. Which medication classification is paired correctly with its physiologic action?

Cholinergic antagonist—causes bronchodilation by inhibiting the parasympathetic nervous system.

The nurse is taking a history from a daughter about her father's onset of stroke signs and symptoms. Which statement by the daughter indicates that the client likely had an embolic stroke?

Client has a long history of atrial fibrillation.

A nurse is caring for a client who had a modified uvulopalatopharyngoplasty (modUPPP) earlier in the day for obstructive sleep apnea. Which assessment finding indicates that a priority goal has been met?

Client is able to swallow own secretions without drooling.

A hospital prepares to receive large numbers of casualties from a community disaster. Which clients would the nurse identify as appropriate for discharge or transfer to another facility? (Select all that apply.)

Client on the medical unit for wound care, Client who had open reduction and internal fixation of a femur fracture 3 days ago

The nurse is caring for four clients with traumatic brain injuries. Which client would the nurse assess first?

Client who has a temperature of 102° F (38.9° C)

A nurse is volunteering in a temporary shelter for survivors of a hurricane. Which client does the nurse anticipate has the highest need for further assessment and referral?

Client with a score of 48 on the Impact of Event Scale-Revised (IES-R)

A nurse is planning discharge teaching on tracheostomy care for an older client. What factors does the nurse need to assess before teaching this particular client? (Select all that apply.)

Cognition, Dexterity, Range of motion , Vision

An emergency department nurse is caring for a client who has died from a suspected homicide. Which action does the nurse take?

Communicate the client's death to the family in a simple and concrete manner.

A nurse is caring for a client with a nonhealing arterial lower leg ulcer. What action by the nurse is best?

Consult with the wound care nurse.

Assess the client's serum sodium level.

Contact the local organ procurement organization as soon as possible.

A client is taking ethambutol for tuberculosis. What instructions does the nurse provide the client regarding this drug? (Select all that apply.)

Contact the primary health care provider if preexisting gout becomes worse. , Report any changes in vision immediately to the health care provider. , You will take this medication along with some others for 8 weeks. , Take this medicine with a full glass of water.

A client is being discharged home after having a tracheostomy placed. What suggestions does the nurse offer to help the client maintain self-esteem? (Select all that apply.)

Create a communication system. , Try loose-fitting shirts with collars. , Wear fashionable scarves.

A nurse cares for a client with congestive heart failure who has a regular cardiac rhythm of 128 beats/min. For which physiologic alterations would the nurse assess? (Select all that apply.)

Decrease in cardiac output , Decrease in blood pressure , Decrease in urine output

The nurse is caring for a client who had a hemorrhagic stroke. Which assessment finding is the earliest sign of increasing intracranial pressure (ICP) for this client?

Decreased level of consciousness

A nurse assesses a client who is recovering from an open traditional anterior cervical fusion. Which assessment findings would alert the nursing to a complication from this procedure? (Select all that apply.)

Difficulty swallowing , Hoarse voice

A client presents to the emergency department with a thoracic aortic aneurysm. Which findings are most consistent with this condition? (Select all that apply.)

Difficulty swallowing , Hoarseness

A client is receiving IV alteplase and reports a sudden severe headache. What is the nurse's first action?

Discontinue the infusion of the drug.

A client with chronic kidney disease (CKD) is refusing to take his medication and has missed two hemodialysis appointments. What is the best initial action for the nurse?

Discuss what the treatment regimen means to the client.

A nurse is providing health education at a community center. Which instructions does the nurse include in teaching about prevention of lightning injuries during a storm? (Select all that apply.)

Do not take a bath or shower., Turn off the television., Put down golf clubs or gardening tools., Seek shelter inside a building or vehicle.

A trauma client with multiple open wounds is brought to the emergency department in cardiac arrest. Which action would the nurse take prior to providing advanced cardiac life support?

Don personal protective equipment.

A client is admitted with a confirmed left middle cerebral artery occlusion. Which assessment findings will the nurse expect? (Select all that apply.)

Dysphagia, Aphasia , Apraxia , Hemiparesis/hemiplegia , Ptosis

A nurse assesses a client with mitral valve stenosis. What clinical sign or symptom would alert the nurse to the possibility that the client's stenosis has progressed?

Dyspnea on exertion

An emergency department nurse assesses a client admitted after a lightning strike. The client is awake but somewhat confused. Which assessment would the nurse complete first?

Electrocardiogram (ECG)

A client is admitted with acute kidney injury (AKI) and a urine output of 2000 mL/day. What is the major concern of the nurse regarding this patient's care?

Electrolyte and fluid imbalance

The nurse is reviewing risk factors in a client who has atherosclerosis. Which findings are most concerning? (Select all that apply.)

Elevated low-density lipoprotein (LDL-C) , Decreased levels of high-density lipoprotein cholesterol (HDL-C, History of smoking

A nurse assesses a client who is prescribed fluticasone and notes oral lesions. What action would the nurse take?

Encourage oral rinsing after fluticasone administration.

A client with early-stage Alzheimer disease is admitted to the hospital with chest pain. Which nursing action is most appropriate to manage this client's dementia?

Ensure a structured and consistent environment.

A nurse prepares to defibrillate a client who is in ventricular fibrillation. Which intervention is appropriate for the nurse to perform prior to defibrillating this client?

Ensure that everyone is clear of contact with the client and the bed.

A client is scheduled to have a tracheostomy placed in an hour. What action by the nurse is the priority?

Ensure that informed consent is on the chart.

A nurse prepares to discharge a client who has heart failure. Based on national quality measures, what actions would the nurse complete prior to discharging this client? (Select all that apply.)

Ensure that the client is prescribed a beta blocker. , Document a discussion about advanced directives., Confirm that a post-discharge nurse visit has been scheduled. , Care transition record transmitted to next level of care within 7 days of discharge.

The nurse initiates care for a client with a cervical spinal cord injury who arrives via emergency medical services. What action would the nurse take first?

Evaluate respiratory status.

A nurse cares for a client after radiation therapy for neck cancer. The client reports extremely dry mouth. What action by the nurse is most appropriate?

Explain that xerostomia may be a permanent side effect.

A nurse assesses a client with multiple sclerosis after administering prescribed fingolimod. For which common side effect would the nurse monitor?

Facial flushing

A nurse is caring for a group of stroke patients. Which clients would the nurse consider referring to a mental health provider? (Select all that apply.)

Female client who exhibits extreme emotional lability , Male client with an initial National Institutes of Health (NIH) Stroke Scale score of 38 , Female client with mild forgetfulness and a history of depression , Male client who has a past hospitalization for a suicide attempt , Male client who is unable to walk or eat 3 weeks poststroke

The nurse assesses a client who has Parkinson disease. Which signs and symptoms would the nurse recognize as a key feature of this disease? (Select all that apply.)

Flexed trunk , Uncontrolled drooling , Slow movements

An emergency department nurse assesses a client who has been raped. With which health care team member would the nurse collaborate when planning this client's care?

Forensic nurse examiner

The nurse is caring for a 60-year-old female client who sustained a thoracic spinal cord injury 10 years ago. For which potential complication will the nurse assess during this client's care?

Fracture

The nurse is caring for four hypertensive clients. Which drug-laboratory value combination would the nurse report immediately to the health care provider?

Furosemide/potassium: 2.1 mEq/L

A nurse reviews laboratory results for a client with diabetes mellitus who is prescribed an intensified insulin regimen:· Fasting blood glucose: 75 mg/dL (4.2 mmol/L)· Postprandial blood glucose: 200 mg/dL (11.1 mmol/L)· Hemoglobin A1C level: 5.5%How would the nurse interpret these laboratory findings?

Good control of blood glucose

A nurse is assessing a client with lung cancer. What nonpulmonary signs and symptoms would the nurse be aware of? (Select all that apply.)

Gynecomastia in male patients , Frequent shaking and sweating relieved by eating , "Moon" face and "buffalo" hump , General edema

The nurse plans care for a client with epilepsy who is admitted to the hospital. Which interventions would the nurse include in this client's plan of care? (Select all that apply.)

Have suction equipment with an airway at the bedside. , Have oxygen administration set at the bedside. , Ensure that the client has IV access.

The nurse is assessing a client who has symptoms of stroke. What are the leading causes of a stroke for which the nurse would assess for this client? (Select all that apply.)

Heavy alcohol intake , Diabetes mellitus , Elevated cholesterol , Obesity , Smoking , Hypertension

A nurse evaluates laboratory results for a client with heart failure. Which results would the nurse expect? (Select all that apply.)

Hematocrit: 32.8% , Serum sodium: 130 mEq/L (130 mmol/L) , Proteinuria, Microalbuminuria

The nurse is caring for a client with increasing intracranial pressure (ICP) following a stroke. Which evidence-based nursing actions are indicated for this client? (Select all that apply.)

Hyperoxygenate the client before and after suctioning. , Avoid sudden or extreme hip or neck flexion. , Provide oxygen to maintain an SaO2 of 95% or greater. , Avoid clustering care nursing activities and procedures.

A nurse is teaching a community group about the long-term effects of untreated sleep apnea. What information does the nurse include? (Select all that apply.)

Hypertension, Stroke , Weight gain , Diabetes, Cognitive deficits , Pulmonary disease

A diabetic client becomes septic after a bowel resection and is having problems with respiratory distress. The nurse reviews the labs and finds the following ABG results: pH 7.50, PaCO2 30, HCO3−: 24, and PaO2 68. What does the nurse recognize as the primary factor causing this the acid-base imbalance?

Hyperventilation due to poor oxygenation

The nurse is caring for a client in late-stage Alzheimer disease. Which assessment finding(s) will the nurse anticipate? (Select all that apply.)

Immobile, ADL dependent , Incontinent, Possible seizures

The nurse assesses a client with diabetic ketoacidosis. Which assessment finding would the nurse correlate with this condition?

Increased rate and depth of respiration

A nurse assesses a client with diabetes mellitus who is admitted with an acid-base imbalance. The client's arterial blood gas values are pH 7.36, PaO2 98 mm Hg, PaCO2 33 mm Hg, and HCO3− 18 mEq/L (18 mmol/L). Which sign or symptom does the nurse identify as an example of the client's compensatory mechanisms?

Increased rate and depth of respirations

A client seen in the emergency department reports fever, fatigue, and dry cough but no other upper respiratory symptoms. A chest x-ray reveals mediastinal widening. What action by the nurse is best?

Inform the client that oral antibiotics will be needed for 60 days.

The nurse is caring for a client on the medical-surgical unit who suddenly becomes unresponsive and has no pulse. The cardiac monitor shows the rhythm below:

Initiate cardiopulmonary resuscitation (CPR).

After administering the first dose of captopril to a client with heart failure, the nurse implements interventions to decrease complications. Which intervention is most important for the nurse to implement?

Instruct the client to ask for assistance when rising from bed.

A client in the emergency department is taking rifampin for tuberculosis. The client reports yellowing of the sclera and skin and bleeding after minor trauma. What laboratory results correlate to this condition? (Select all that apply.)

International normalized ratio (INR): 6.3 , Prothrombin time: 35 seconds

A nurse is discharging a client from the emergency department who has a mild traumatic brain injury. What information obtained from the client represents a possible barrier to self-management? (Select all that apply.)

Is allergic to acetaminophen. , Lives alone and is new in town with no friends. , Plans to have a beer and go to bed once home.

A nurse cares for a client who is prescribed an intravenous prostacyclin agent for pulmonary artery hypertension. What actions would the nurse take to ensure the client's safety while on this medication? (Select all that apply.)

Keep an intravenous line dedicated strictly to the infusion. , Ensure that there is always a backup drug cassette available. , Use strict aseptic technique when using the drug delivery system.

A nurse cares for a client who has a pleural chest tube. What action would the nurse take to ensure safe use of this equipment?

Keep padded clamps at the bedside for use if the drainage system is interrupted.

A nurse cares for a client who is infected with Burkholderia cepacia. What action would the nurse take first when admitting this client to a pulmonary care unit?

Keep the client separated from other clients with cystic fibrosis.

A new graduate nurse has started working on a medical-surgical unit. What actions would the nurse take to be prepared for a disaster? (Select all that apply.)

Know the institution's Emergency Response Plan., Participate in the institution's disaster drill., Be willing to be flexible working during a crisis situation., Develop a personal preparedness plan., Understand that nurses play a role in every phase of a disaster.

A nurse is evaluating levels and functions of trauma centers. Which function is appropriately paired with the level of the trauma center?

Level II—located within community hospitals and provides care to most injured clients.

A telemetry nurse assesses a client who has a heart rate of 35 beats/min on the cardiac monitor. Which assessment would the nurse complete next?

Level of consciousness

An emergency department nurse is caring for a client who is homeless. Which action would the nurse take to gain the client's trust?

Listen to the client's concerns and needs.

A client is hospitalized in the oliguric phase of acute kidney injury (AKI) and is receiving tube feedings. The nurse is teaching the client's spouse about the renal-specific formulation for the enteral solution compared to standard formulas. What components would be discussed in the teaching plan? (Select all that apply.)

Lower sodium , Lower potassium , Higher calories

A nurse prepares to discharge a client with a cardiac dysrhythmia who is prescribed home health care services. Which priority information would be communicated to the home health nurse upon discharge?

Medication orders for home

A nurse is assessing clients who are at risk for acid-base imbalance. Which clients are correctly paired with the acid-base imbalance? (Select all that apply.)

Metabolic acidosis—older adult who is following a carbohydrate-free diet, Respiratory alkalosis—client on mechanical ventilation at a rate of 28 breaths/min, Metabolic alkalosis—older client prescribed antacids for gastroesophageal reflux disease

A nurse assesses a client admitted with a brown recluse spider bite. Which assessment does the nurse perform to identify complications of this bite?

Monitor the client's temperature every 4 hours.

A nurse is field-triaging clients after an industrial accident. Which client condition would the nurse triage with a red tag?

Multiple fractured ribs and shortness of breath

The nurse is preparing for discharge of a client who had a carotid artery angioplasty with stenting to prevent a stroke. For which signs and symptoms with the nurse teach the family to report to the primary health care provider immediately? (Select all that apply.)

Muscle weakness , Hoarseness, Acute confusion , Severe headache , Dysphagia

An elderly client who has fallen from a roof is transported to the emergency department by ambulance. The client was unconscious at the scene but is conscious on arrival and is triaged as urgent. What is the priority assessment the nurse includes during the primary survey of the patient?

Neurologic status

A client is placed on fluid restriction because of chronic kidney disease (CKD). Which assessment finding would alert the nurse that the client's fluid balance is stable at this time?

No adventitious sounds in the lungs

A client is receiving an infusion of alteplase for an intra-arterial clot. The client begins to mumble and is disoriented. What action by the nurse is most important?

Notify the Rapid Response Team.

A client is taking furosemide 40 mg/day for management of early chronic kidney disease (CKD). To assess the therapeutic effect of the medication, what action of the nurse is best?

Obtain daily weights of the client.

The nurse is teaching a group of college students about the importance of preventing meningitis. Which health promotion activity is the most appropriate for preventing this disease?

Obtaining the recommended meningitis vaccination and boosters

A nurse cares for a client with a spinal cord injury. With which interprofessional health team member would the nurse collaborate to assist the client with activities of daily living?

Occupational therapist

A nurse cares for a client who has developed esophagitis after undergoing radiation therapy for lung cancer. Which diet selection would the nurse provide for this client?

Omelet, soft whole-wheat bread

A nurse assesses a client who is at risk for neck cancer. Which symptoms would the nurse assess for? (Select all that apply.)

Oral mucosa is gray or dark brown , Pain when drinking grapefruit juice , Oral lesions that are over 2 weeks old , Changes in the patient's voice quality

A nurse is caring for a client with a deep vein thrombosis (DVT). What nursing assessment indicates that an important outcome has been met?

Oxygen saturation of 98%

A nurse assesses a client with a spinal cord injury at level T5. The client's blood pressure is 184/95 mm Hg, and the client presents with a flushed face and blurred vision. After raising the head of the bed, what action would the nurse take next?

Palpate the bladder for distention.

A new nurse is caring for a client with an abdominal aneurysm. What action by the new nurse requires the nurse's mentor to intervene?

Palpates the abdomen in four quadrants.

The nurse assesses a client who has meningitis. Which sign(s) and symptom(s) would the nurse anticipate? (Select all that apply.)

Photophobia, Decreased level of consciousness , Severe headache , Fever and chills

A nurse assesses cerebrospinal fluid leaking onto a client's surgical dressing. What actions would the nurse take? (Select all that apply.)

Place the client in a flat position. , Report the leak to the surgeon.

An emergency department (ED) case manager is consulted for a client who is homeless. Which intervention would the case manager provide?

Provide referrals to subsidized community-based health clinics.

A hospital responds to a local mass casualty event. What action would the nurse supervisor take to prevent staff posttraumatic stress disorder during and after the event?

Provide water and healthy snacks for energy throughout the event.

A nurse is preparing a client for a femoropopliteal bypass operation. What actions does the nurse delegate to the assistive personnel (AP)? (Select all that apply.)

Raising the side rails on the bed , Recording baseline vital signs

A nurse assesses a client who is experiencing an acid-base imbalance. The client's arterial blood gas values are pH 7.32, PaO294 mm Hg, PaCO2 34 mm Hg, and HCO3− 18 mEq/L (18 mmol/L). For which clinical signs and symptoms would the nurse assess? (Select all that apply.)

Reduced deep tendon reflexes, Drowsiness, Increased respiratory rate, Flaccid paralysis

A nurse receives a hand-off report on a female client who had a left-sided stroke with homonymous hemianopsia. What action by the nurse is most appropriate for this client?

Remind the client to move her head from side to side to increase her visual field.

A nurse assesses a client with paraplegia from a spinal cord injury and notes reddened areas over the client's hips and sacrum. What actions would the nurse take? (Select all that apply.)

Reposition the client off of the reddened areas. , Apply a pressure-reducing mattress.

A client who has multiple sclerosis reports increased severe muscle spasticity and tremors. What nursing action is most appropriate to manage this client's concern?

Request a prescription for an antispasmodic drug such as baclofen.

An emergency department (ED) nurse is preparing to transfer a client to the trauma intensive care unit. Which information would the nurse include in the nurse-to-nurse hand-off report? (Select all that apply.)

Safety concerns, List of home medications, Diagnostic test results, Mechanism of injury

A nurse cares for a client with atrial fibrillation who reports fatigue when completing activities of daily living. What intervention would the nurse implement to address this client's concerns?

Schedule periods of exercise and rest during the day.

The nurse is taking a history on an older adult. Which factors would the nurse assess as potential risks for low back pain? (Select all that apply.)

Scoliosis , Spinal stenosis , Hypocalcemia, Osteoporosis , Osteoarthritis

A nurse prepares to discharge an older adult client home from the emergency department (ED). What actions does the nurse take to prevent future ED visits? (Select all that apply.)

Screen for depression and suicide., Complete a functional assessment.

The nurse assesses a client who has a mild traumatic brain injury (TBI) for signs and symptoms consistent with this injury. What signs and symptoms does the nurse expect? (Select all that apply.)

Sensitivity to light and sound , Reports "feeling foggy"

A nurse reviews the laboratory results of a client who is receiving intravenous insulin. Which would alert the nurse to intervene immediately?

Serum potassium level of 2.5 mEq/L (2.5 mmol/L)

A nurse administers prescribed adenosine to a client. Which response would the nurse assess for as the expected therapeutic response?

Short period of asystole

A nurse is caring for a client with acute pericarditis who reports substernal precordial pain that radiates to the left side of the neck. Which nonpharmacologic comfort measure would the nurse implement?

Sit the client up with a pillow to lean forward on.

The nurse is collaborating with the occupational therapist to assist a client with a complete cervical spinal cord injury to transfer from the bed to the wheelchair. What ambulatory aid would be most appropriate for the client to meet this outcome?

Sliding board

A nurse is teaching a client who has premature ectopic beats. Which education would the nurse include in this client's teaching? (Select all that apply.)

Smoking cessation , Stress reduction and management , Adverse effects of medications

A nurse is caring for a client who is experiencing excessive diarrhea. The client's arterial blood gas values are pH 7.18, PaO2 98 mm Hg, PaCO2 45 mm Hg, and HCO3− 16 mEq/L (16 mmol/L). Which primary health care provider order does the nurse expect to receive?

Sodium bicarbonate

A nurse cares for a client with infective endocarditis. Which infection control precautions would the nurse use?

Standard Precautions

On a hot humid day, an emergency department nurse is caring for a client who is confused and has these vital signs: temperature 104.1° F (40.1° C), pulse 132 beats/min, respirations 26 breaths/min, and blood pressure 106/66 mm Hg. What action would the nurse take?

Start an intravenous line and infuse 0.9% saline solution.

A nurse is constructing a personal preparedness plan in case of a disaster. What does the nurse consider in making this plan?

Store basic supplies to last for at least 3 days.

The emergency department nurse is participating in a bioterrorism drill in which several "clients" are suspected to have inhalation anthrax. Which "clients" would the nurse see as the priorities? (Select all that apply.)

Stridor , Oxygen saturation of 91% , Diaphoresis

A nurse provides diabetic education at a public health fair. Which disorders would the nurse include as complications of diabetes mellitus? (Select all that apply.)

Stroke , Kidney failure , Blindness

A nurse teaches a client who had a supraglottic laryngectomy. Which technique would the nurse teach the client to prevent aspiration?

Swallow twice while bearing down.

A nurse is caring for a client who is experiencing moderate metabolic alkalosis. What action would the nurse take?

Teach the client fall prevention measures.

A client is in the family practice clinic reporting a severe "cold" that started 4 days ago. On examination, the nurse notes that the client also has a severe headache and muscle aches. What action by the nurse is best?

Teach the client to sneeze in the upper sleeve.

A nurse wants to become involved in community disaster preparedness and is interested in helping setup and staff first-aid stations or community acute care centers in the event of a disaster. Which organization is the best fit for this nurse's interests?

The Medical Reserve Corps

A home health nurse is visiting a new client who uses oxygen in the home. For which factors does the nurse assess when determining if the client is using the oxygen safely? (Select all that apply.)

The client does not allow smoking in the house. , Electrical cords are in good working order. , Flammable liquids are stored in the garage.

A client with a new tracheostomy is being seen in the oncology clinic. What finding by the nurse best indicates that goals for the client's decrease in self-esteem are being met?

The client has joined a book club that meets at the library.

After teaching a client how to perform diaphragmatic breathing, the nurse assesses the client's understanding. Which action demonstrates that the client correctly understands the teaching?

The client places his or her hands on the abdomen.

The nurse is preparing to administer IV alteplase for a client diagnosed with an acute ischemic stroke. Which statement is correct about the administration of this drug?

The drug is not given to clients who are already on anticoagulant or antiplatelet therapy.

A client, who has become increasingly dyspneic over a year, has been diagnosed with pulmonary fibrosis. What information would the nurse plan to include in teaching this client? (Select all that apply.)

The need to avoid large crowds and people who are ill , Safety measures to take if home oxygen is needed., Information about appropriate use of the drug nintedanib , Measures to avoid fatigue during the day

The nurse assesses a client with a history of epilepsy who experiences stiffening of the muscles of the arms and legs, followed by an immediate loss of consciousness and jerking of all extremities. How would the nurse document this type of seizure?

Tonic-clonic

While triaging clients in a crowded emergency department, a nurse assesses a client who presents with symptoms of tuberculosis. Which action would the nurse take first?

Transfer the client to a negative-pressure room.

A nurse assists with the cardioversion of a client experiencing acute atrial fibrillation. What action would the nurse take prior to the cardioversion?

Turn off oxygen therapy.

The nurse witnesses a client begin to experience a tonic-clonic seizure and loss of consciousness. What action would the nurse take first?

Turn the client's head to the side.

An emergency department manager wants to mitigate the possible acute and chronic stress after mass casualty events in the staff. What action would the manager take?

Use available resources for broad education and training in disaster management.

A nurse is caring for clients in a busy emergency department. What actions would the nurse take to ensure client and staff safety? (Select all that apply.)

Use facility policy identification procedures for "Jane/John Doe" clients., Check clients for a medical alert bracelets or necklaces., Search the belongings of clients with altered mental status to gain essential medical information., Attempt de-escalation strategies for clients who demonstrate aggressive behaviors., Use two identifiers before each intervention and before mediation administration.

The nurse assesses a client who has a history of migraines. Which symptom would the nurse identify as an early sign of a migraine with aura?

Visual disturbances

A nurse evaluates prescriptions for a client with chronic atrial fibrillation. Which medication would the nurse expect to find on this client's medication administration record to prevent a common complication of this condition?

Warfarin

A client calls the clinic to report exposure to poison ivy and an itchy rash that is not helped with over-the-counter antihistamines. What response by the nurse is mostappropriate? Select one: a. "Antihistamines do not help poison ivy." b. "There are different antihistamines to try." c. "You should be seen in the clinic right away." d. "You will need to take some IV steroids."

a. "Antihistamines do not help poison ivy."

A new nurse reports to the nurse preceptor that a client requested pain medication, and when the nurse brought it, the client was sound asleep. The nurse states the client cannot possibly sleep with the severe pain the client described. Which response by the experienced nurse is best? a. "Being able to sleep doesn't mean pain doesn't exist." b. "Have you ever experienced any type of pain?" c. "You're right; I would put the medication back." d. "The client should be assessed for drug addiction."

a. "Being able to sleep doesn't mean pain doesn't exist."

A nurse is preparing to administer IV chemotherapy. What supplies does this nurse need? (Select all that apply.) Select one or more: a. "Chemo" gloves b. Face mask c. Impervious gown d. N95 respirator e. Shoe covers f. Eye protection

a. "Chemo" gloves b. Face mask c. Impervious gown f. Eye protection

A nurse teaches a client who is at risk for carpal tunnel syndrome. Which health promotion activities would the nurse include in the health teaching? (Select all that apply.) Select one or more: a. "Frequently assesses the ergonomics of the equipment being used." b. "Take breaks to stretch fingers and wrists during working hours." c. "Do not participate in activities that require repetitive actions." d. "Take ibuprofen to decrease pain and swelling in wrists." e. "Adjust chair height to allow for good posture."

a. "Frequently assesses the ergonomics of the equipment being used." b. "Take breaks to stretch fingers and wrists during working hours." e. "Adjust chair height to allow for good posture."

While assessing a client's peripheral IV site, the nurse observes a streak of red along the vein path and palpates a 1.5 inch (4-cm) venous cord. How will the nurse document this finding? Select one: a. "Grade 3 phlebitis at IV site" b. "Infection at IV site" c. "Thrombosed area at IV site" d. "Infiltration at IV site"

a. "Grade 3 phlebitis at IV site"

After teaching a client who is recovering from a vertebroplasty, the nurse assesses the patient's understanding. Which statement by the client indicates a need for additional teaching? Select one: a. "I can drive myself home after the procedure." b. "I will monitor the puncture site for signs of infection." c. "I can start walking tomorrow and increase my activity slowly." d. "I will remove the dressing the day after discharge."

a. "I can drive myself home after the procedure."

A nurse has educated a client on an epinephrine autoinjector. What statement by the client indicates additional instruction is needed? Select one: a. "I don't need to go to the hospital after using it." b. "I must carry two autoinjectors with me at all times." c. "I will write the expiration date on my calendar." d. "This can be injected right through my clothes."

a. "I don't need to go to the hospital after using it."

A new nurse is caring for a client receiving drug therapy via a smart pump. What statement by the new nurse demonstrates the need for more instruction on this technology? Select one: a. "I don't need to manually calculate IV infusion rates with smart pumps." b. "Responding to IV pump alarms is a high priority for client safety." c. "The hospital can preprogram the pumps for high-alert drug limits." d. "These pumps have a system to prevent fluids from free-flowing into the client."

a. "I don't need to manually calculate IV infusion rates with smart pumps."

A nurse is assessing coping in older women in a support group for recent widows. Which statement by a participant best indicates potential for successful coping? a. "I have had the same best friend for decades." b. "My kids come to see me every weekend." c. "I think I am coping very well on my own." d. "Oh, I have lots of friends at the senior center."

a. "I have had the same best friend for decades."

The nurse has educated a client on precautions to take with thrombocytopenia. What statement by the client indicates a need to review the information? Select one: a. "I will be careful if I need enemas for constipation." b. "I will use an electric shaver instead of a razor." c. "I should only eat soft food that is either cool or warm." d. "I won't be able to play sports with my grandkids."

a. "I will be careful if I need enemas for constipation."

A nurse teaches a client about performing intermittent self-catheterization. The client states, "I am not sure if I will be able to afford these catheters." How would the nurse respond? a. "I will contact the social worker who will discuss potential resources with you." b. "Instead of purchasing new catheters, you can boil the catheters and reuse them up to 10 times each." c. "Even though it is expensive, the cost of taking care of urinary tract infections would be even higher." d. "I will try to find out whether you qualify for money to purchase these necessary supplies."

a. "I will contact the social worker who will discuss potential resources with you."

The nurse is caring for a client who is planning to have a laparoscopic colon resection for colorectal cancer tomorrow. Which statement by the client indicates a need for further teaching? a. "I will probably be in the hospital for 3 to 4 days after surgery." b. "I will be able to return to work in a week or two depending on how I do." c. "I will be able to walk around a little on the same day as the surgery." d. "I should have less pain after this surgery compared to having a large incision."

a. "I will probably be in the hospital for 3 to 4 days after surgery."

The nurse is teaching a client who is prescribed acetaminophen for control of osteoarthritic joint pain. What statement by the client indicates a need for further teaching? Select one: a. "I won't take more than 5000 mg of this drug each day." b. "I'll follow up to get my lab tests done to check my liver." c. "I'll check drugs that I take for acetaminophen in them." d. "I can use topical patches and creams to help relieve pain."

a. "I won't take more than 5000 mg of this drug each day."

The nurse teaches a client about how to prevent transmission of gastroenteritis. Which statement by the nurse indicates a need for further teaching? a. "I'll cook all the meals for my family." b. "I'll keep my bathroom extra clean." c. "I won't let anyone use my dishes or glasses." d. "I'll wash my hands with antibacterial soap."

a. "I'll cook all the meals for my family."

An assistive personnel asks why brushing client s' teeth with a toothbrush in the intensive care unit is important to infection control. What response by the registered nurse is best? Select one: a. "It mechanically removes biofilm on teeth." b. "It's easier to clean all surfaces with a brush." c. "Oral care is important to all our clients." d. "Toothbrushes last longer than oral swabs."

a. "It mechanically removes biofilm on teeth."

A nurse cares for a client who states, "My husband is repulsed by my colostomy and refuses to be intimate with me." How would the nurse respond? a. "Let's talk to the ostomy nurse to help you and your husband work through this." b. "You could try to wear longer lingerie that will better hide the ostomy appliance." c. "If you are not careful, you can hurt the stoma if you engage in sexual activity." d. "You should empty the pouch first so it will be less noticeable for your husband."

a. "Let's talk to the ostomy nurse to help you and your husband work through this."

A client asks the nurse if eating only preservative- and dye-free foods will decrease cancer risk. What response by the nurse is best? Select one: a. "Maybe; preservatives, dyes, and preparation methods may be risk factors." b. "No; research studies have never shown those things to cause cancer." c. "There are other things you can do that will more effectively lower your risk." d. "Yes; preservatives and dyes are well known to be carcinogens."

a. "Maybe; preservatives, dyes, and preparation methods may be risk factors."

The nurse is teaching assistive personnel about postoperative care for an older adult who had a posterolateral total hip arthroplasty. What teaching will the nurse include? (Select all that apply.) Select one or more: a. "Move the client slowly to prevent dizziness and a possible fall." b. "Encourage the client to deep breathe and cough at least every 2 hours." c. "Help the client use the incentive spirometer at least every 2 hours." d. "Keep the abduction pillow in place at all times while the client is in bed." e. "Let me know if the client has an elevated temperature or pulse." f. "Keep in mind that the client may be a little confused after surgery." g. "Please let me know if you see any reddened or open skin areas during bathing."

a. "Move the client slowly to prevent dizziness and a possible fall." b. "Encourage the client to deep breathe and cough at least every 2 hours." c. "Help the client use the incentive spirometer at least every 2 hours." d. "Keep the abduction pillow in place at all times while the client is in bed." e. "Let me know if the client has an elevated temperature or pulse." f. "Keep in mind that the client may be a little confused after surgery."

The new nurse asks the preceptor how context affects clinical judgment. What response by the preceptor is best? a. "Outside influences such as environment in which you provide care, influence your decisions." b. "The context of the situation provides an extra layer of complexity to consider." c. "Context considers the whole of the patient's story and circumstances." d. "It shouldn't, only nursing knowledge would affect clinical judgment."

a. "Outside influences such as environment in which you provide care, influence your decisions."

A rehabilitation nurse is caring for an older adult client who states, "I tire easily." How would the nurse respond? (Select all that apply.) a. "Schedule all of your tasks for the morning when you have the most energy." b. "Try to break large activities into smaller parts to allow rest periods between activities." c. "Your family could hire someone who can assist you with daily chores." d. "Plan to gather all of the supplies needed for a chore prior to starting the activity." e. "Try to rest before and after eating or going to the bathroom."

a. "Schedule all of your tasks for the morning when you have the most energy." b. "Try to break large activities into smaller parts to allow rest periods between activities." d. "Plan to gather all of the supplies needed for a chore prior to starting the activity." e. "Try to rest before and after eating or going to the bathroom."

A nurse assesses a client who is admitted to the inpatient rehabilitation unit with hip problems. The client asks, "Why are you asking about my bowels and bladder?" How would the nurse respond? a. "To plan your care based on your normal elimination routine." b. "So we can help prevent side effects of your medications." c. "To schedule your activities around your elimination pattern." d. "We need to evaluate your ability to function independently."

a. "To plan your care based on your normal elimination routine."

A nurse prepares to discharge a client who is newly diagnosed with a chronic inflammatory bowel disease. Which questions would the nurse ask in preparation for discharge? (Select all that apply.) a. "When should you contact your provider?" b. "Do you have a scale for daily weights?" c. "How many bathrooms are in your home?" d. "Does your gym provide yoga classes?" e. "What do you plan to eat for dinner?"

a. "When should you contact your provider?" c. "How many bathrooms are in your home?" d. "Does your gym provide yoga classes?" e. "What do you plan to eat for dinner?"

A nurse assesses clients at a community health center. Which client is at highest risk for developing colorectal cancer? a. A 72-year-old who eats fast food frequently. b. A 44-year-old with irritable bowel syndrome (IBS). c. A 37-year-old who drinks eight cups of coffee daily. d. A 60-year-old lawyer who works 65 hours per week.

a. A 72-year-old who eats fast food frequently.

A nurse is providing community education on the seven warning signs of cancer. Which signs are included? (Select all that apply.) Select one or more: a. A sore that does not heal b. Changes in menstrual patterns c. Indigestion or trouble swallowing d. Near-daily abdominal pain e. Obvious change in a mole f. Frequent indigestion

a. A sore that does not heal b. Changes in menstrual patterns c. Indigestion or trouble swallowing e. Obvious change in a mole f. Frequent indigestion

A nurse is teaching a client newly diagnosed with osteoarthritis (OA) about drugs used to treat the disease. For which drug does the nurse plan health teaching? Select one: a. Acetaminophen b. Cyclobenzaprine hydrochloride c. Hyaluronate d. Ibuprofen

a. Acetaminophen

A nurse learns the concepts of addiction, tolerance, and dependence. Which information is accurate? (Select all that apply.) a. Addiction is a chronic physiologic disease process. b. Tolerance is a normal response to regular opioid use. c. Tolerance is said to occur when opioid effects decrease. d. Physical dependence occurs after repeated doses of an opioid. e. Physical dependence and addiction are the same thing. f. Pseudoaddiction can result in withdrawal symptoms.

a. Addiction is a chronic physiologic disease process. b. Tolerance is a normal response to regular opioid use. c. Tolerance is said to occur when opioid effects decrease. d. Physical dependence occurs after repeated doses of an opioid.

A client with chronic osteomyelitis is being discharged from the hospital. What information is important for the nurse to teach this client and family? (Select all that apply.) Select one or more: a. Adherence to the antibiotic regimen b. Correct intramuscular injection technique c. Eating high-protein and high-carbohydrate foods d. Keeping daily follow-up appointments e. Proper use of the intravenous equipment

a. Adherence to the antibiotic regimen c. Eating high-protein and high-carbohydrate foods e. Proper use of the intravenous equipment

A primary health care provider notifies the nurse that a client has a "bandemia." What action does the nurse anticipate? Select one: a. Administer antibiotics. b. Place the client in isolation. c. Administer IV leukocytes. d. Obtain an immunization history.

a. Administer antibiotics.

A client is to receive a fecal microbiota transplantation tomorrow (FMT). What action by the nurse is best? Select one: a. Administer bowel cleansing as prescribed. b. Educate the client on immunosuppressive drugs. c. Inform the client he/she will drink a thick liquid. d. Place a nasogastric tube to intermittent suction.

a. Administer bowel cleansing as prescribed.

A client has a metastatic bone tumor in the left leg. What action by the nurse is appropriate? Select one: a. Administer pain medication as prescribed. b. Elevate the extremity and apply moist heat. c. Teach the client about amputation care. d. Place the client on protective precautions.

a. Administer pain medication as prescribed.

A client is being admitted with suspected tuberculosis (TB). What actions by the nurse are best? (Select all that apply.) Select one or more: a. Admit the client to a negative-airflow room. b. Maintain a distance of 3 feet (1 m) from the client at all times. c. Obtain specialized respirators for caregiving. d. Other than wearing gloves, no special actions are needed. e. Wash hands with chlorhexidine after providing care. f. Assure client has a respirator for moving between departments.

a. Admit the client to a negative-airflow room. c. Obtain specialized respirators for caregiving.

The nurse learns that which risk factors can affect immunity? (Select all that apply.) Select one or more: a. Age b. Environmental factors c. Ethnicity d. Drugs e. Nutritional status

a. Age b. Environmental factors d. Drugs e. Nutritional status

A nurse asks the supervisor why older adults are more prone to infection than other adults. What reasons does the supervisor give? (Select all that apply.) Select one or more: a. Age-related decrease in immune function b. Decreased cough and gag reflexes c. Diminished acidity of gastric secretions d. Increased lymphocytes and antibodies e. Thinning skin that is less protective f. Higher rates of chronic illness

a. Age-related decrease in immune function b. Decreased cough and gag reflexes c. Diminished acidity of gastric secretions e. Thinning skin that is less protective f. Higher rates of chronic illness

A nurse is assessing a community group for dietary factors that contribute to osteoporosis. In addition to inquiring about calcium, the nurse also assesses for which other dietary components? (Select all that apply.) Select one or more: a. Alcohol b. Caffeine c. Fat d. Carbonated beverages e. Vitamin D

a. Alcohol b. Caffeine d. Carbonated beverages e. Vitamin D

The nurse is presenting information to a community group on safer sex practices. The nurse would teach that which sexual practice is the riskiest? Select one: a. Anal intercourse b. Masturbation c. Oral sex d. Vaginal intercourse

a. Anal intercourse

Which are steps in the process of making an antigen-specific antibody? (Select all that apply.) Select one or more: a. Antibody-antigen binding b. Invasion c. Opsonization d. Recognition e. Sensitization f. Production

a. Antibody-antigen binding b. Invasion d. Recognition e. Sensitization f. Production

For a person to be immunocompetent, which processes need to be functional and interact appropriately with each other? (Select all that apply.) Select one or more: a. Antibody-mediated immunity b. Cell-mediated immunity c. Inflammation d. Red blood cells e. White blood cells

a. Antibody-mediated immunity b. Cell-mediated immunity c. Inflammation

A client receiving radiation therapy reports severe skin itching and irritation. What actions does the nurse delegate to assistive personnel (AP)? (Select all that apply.) Select one or more: a. Apply approved moisturizers to dry skin. b. Apply steroid creams to the skin. c. Bathe the client using mild soap. d. Help the client pat skin dry after a bath. e. Teach the client to avoid sunlight. f. Make sure no clothing is rubbing the site.

a. Apply approved moisturizers to dry skin. c. Bathe the client using mild soap. d. Help the client pat skin dry after a bath. f. Make sure no clothing is rubbing the site.

The nurse is caring for a client who just had a minimally invasive inguinal hernia repair. Which nursing actions would the nurse implement? (Select all that apply.) a. Apply ice to the surgical area for the first 24 hours after surgery. b. Teach the client to rest for several days after surgery when at home. c. Encourage deep breathing after surgery but teach the client to avoid coughing. d. Teach the client not to lift more than 10 lb (4.5 kg) until allowed by the surgeon. e. Encourage ambulation with assistance within the first few hours after surgery f. Assess vital signs frequently for the first few hours after surgery.

a. Apply ice to the surgical area for the first 24 hours after surgery. b. Teach the client to rest for several days after surgery when at home. c. Encourage deep breathing after surgery but teach the client to avoid coughing. d. Teach the client not to lift more than 10 lb (4.5 kg) until allowed by the surgeon. e. Encourage ambulation with assistance within the first few hours after surgery f. Assess vital signs frequently for the first few hours after surgery.

A client has thrombocytopenia. What actions does the nurse delegate to assistive personnel (AP)? (Select all that apply.) Select one or more: a. Apply the client's shoes before getting the client out of bed. b. Assist the client with ambulation. c. Shave the client with a safety razor only. d. Use a lift sheet to move the client up in bed.

a. Apply the client's shoes before getting the client out of bed. b. Assist the client with ambulation. d. Use a lift sheet to move the client up in bed.

The nurse learns that effective antimicrobial therapy requires which factors to be present? (Select all that apply.) Select one or more: a. Appropriate drug b. Proper route of administration c. Standardized peak levels d. Sufficient dose e. Sufficient length of treatment f. Appropriate trough levels

a. Appropriate drug b. Proper route of administration d. Sufficient dose e. Sufficient length of treatment

A client's family members are concerned that telling the client about a new finding of cancer will cause extreme emotional distress. They approach the nurse and ask if this can be kept from the client. What actions by the nurse are most appropriate?(Select all that apply.) Select one or more: a. Ask the family to describe their concerns more fully. b. Consult with a social worker, chaplain, or ethics committee. c. Explain the client's right to know and ask for their assistance. d. Have the unit manager take over the care of this client and family. e. Tell the family that this secret will not be kept from the client.

a. Ask the family to describe their concerns more fully. b. Consult with a social worker, chaplain, or ethics committee. c. Explain the client's right to know and ask for their assistance.

A client with HIV-III is hospitalized with P. jiroveci pneumonia and is started on the drug of choice for this infection. What laboratory values would be most important for the nurse report to the primary health care provider? (Select all that apply.) Select one or more: a. Aspartate transaminase, alanine transaminase: elevated b. CD4+ cell count: 180/mm3 c. Creatinine: 1.0 mg/dL (88 mcmol/L) d. Platelet count: 80,000/mm3 (80 × 109/L) e. Serum sodium: 120 mEq/L (120 mmol/L) f. Serum potassium: 3.4 mEq/L (3.4 mmol/L)

a. Aspartate transaminase, alanine transaminase: elevated d. Platelet count: 80,000/mm3 (80 × 109/L) e. Serum sodium: 120 mEq/L (120 mmol/L)

A client receiving chemotherapy has a white blood cell count of 1000/mm3 (1 × 109/L). What actions by the nurse are most appropriate?(Select all that apply.) Select one or more: a. Assess all mucous membranes every 4 to 8 hours. b. Do not allow the client to eat meat or poultry. c. Listen to lung sounds and monitor for cough. d. Monitor the venous access device appearance hourly. e. Take and record vital signs every 4 to 8 hours. f. Encourage activity the client can tolerate.

a. Assess all mucous membranes every 4 to 8 hours. c. Listen to lung sounds and monitor for cough. d. Monitor the venous access device appearance hourly. e. Take and record vital signs every 4 to 8 hours.

A nurse uses the Checklist of Nonverbal Pain Indicators to assess pain in a nonverbal client with advanced dementia but no other medical history except well-controlled hypertension and high cholesterol. The client scores a zero. Which action by the nurse is best? a. Assess physiologic indicators and vital signs. b. Do not give pain medication as no pain is indicated. c. Document the findings and continue to monitor. d. Try a small dose of analgesic medication for pain.

a. Assess physiologic indicators and vital signs.

A nurse is assessing an older client for the presence of infection. The client's temperature is 97.6° F (36.4° C). What response by the nurse is best? Select one: a. Assess the client for more specific signs. b. Conclude that an infection is not present. c. Document findings and continue to monitor. d. Request the primary health care provider order blood cultures

a. Assess the client for more specific signs.

A nurse is assessing an older client for the presence of infection. The client's temperature is 97.6° F (36.4° C). What response by the nurse is best? Select one: a. Assess the client for more specific signs. b. Conclude that an infection is not present. c. Document findings and continue to monitor. d. Request the primary health care provider order blood cultures.

a. Assess the client for more specific signs.

A client has just been informed of a positive HIV test. The client is distraught and does not know what to do. What intervention by the nurse is best? Select one: a. Assess the client for support systems. b. Determine if a clergy member would help. c. Explain legal requirements to tell sex partners. d. Offer to tell the family for the client.

a. Assess the client for support systems.

A client with bone cancer is hospitalized for a limb salvage procedure. How can the nurse best address the client's psychosocial needs? Select one: a. Assess the client's coping skills and support systems. b. Explain that the surgery leads to a longer life expectancy. c. Refer the client to the social worker or hospital chaplain. d. Reinforce physical therapy to aid with ambulating normally.

a. Assess the client's coping skills and support systems.

A client with a history of prostate cancer is in the clinic and reports new onset of severe low back pain. What action by the nurse is most important? Select one: a. Assess the client's gait and balance. b. Ask the client about the ease of urine flow. c. Document the report completely. d. Inquire about the client's job risks.

a. Assess the client's gait and balance.

A nurse is assessing a client with hypokalemia, and notes that the client's handgrip strength has diminished since the previous assessment 1 hour ago. What action does the nurse take first? Select one: a. Assess the client's respiratory rate, rhythm, and depth. b. Measure the client's pulse and blood pressure. c. Document findings and monitor the client. d. Call the health care primary health care provider.

a. Assess the client's respiratory rate, rhythm, and depth.

A client is receiving chemotherapy through a peripheral IV line. What action by the nurse is most important? Select one: a. Assessing the IV site and blood return every hour b. Educating the client on side effects c. Monitoring the client for nausea d. Providing warm packs for comfort

a. Assessing the IV site and blood return every hour

A clinic nurse is working with an older client. What action is most important for preventing infections in this client? Select one: a. Assessing vaccination records for booster shot needs b. Encouraging the client to eat a nutritious diet c. Instructing the client to wash minor wounds carefully d. Teaching hand hygiene to prevent the spread of microbes

a. Assessing vaccination records for booster shot needs

A client has mucositis. What actions by the nurse will improve the client's nutrition? (Select all that apply.) Select one or more: a. Assist with rinsing the mouth with saline frequently. b. Encourage the client to eat room-temperature foods. c. Give the client hot liquids to hold in the mouth. d. Provide local anesthetic medications to swish and spit. e. Remind the client to brush teeth gently after each meal. f. Offer the client fluids to drink each hour.

a. Assist with rinsing the mouth with saline frequently. b. Encourage the client to eat room-temperature foods. d. Provide local anesthetic medications to swish and spit. e. Remind the client to brush teeth gently after each meal. f. Offer the client fluids to drink each hour.

A nursing manager is concerned about the number of infections on the hospital unit. What action by the manager would best help prevent these infections? Select one: a. Auditing staff members' hand hygiene practices b. Ensuring clients are placed in appropriate isolation c. Establishing a policy to remove urinary catheters quickly d. Teaching staff members about infection control methods

a. Auditing staff members' hand hygiene practices

A client is receiving rituximab. What assessment by the nurse takes priority? Select one: a. Blood pressure b. Temperature c. Oral mucous membranes d. Pain

a. Blood pressure

A nurse learning about antibody-mediated immunity learns that the cell with the most direct role in this process begins development in which tissue or organ? Select one: a. Bone marrow b. Spleen c. Thymus d. Tonsils

a. Bone marrow

A nurse is learning about human immune deficiency virus (HIV) infection. Which statements about HIV infection are correct? (Select all that apply.) Select one or more: a. CD4+ cells begin to create new HIV virus particles. b. Antibodies produced are incomplete and do not function well. c. Macrophages stop functioning properly. d. Opportunistic infections and cancer are leading causes of death. e. People with HIV-I disease are not infectious to others. f. The CD4+ T-cell is only affected when the disease has progressed to HIV-III

a. CD4+ cells begin to create new HIV virus particles. b. Antibodies produced are incomplete and do not function well. c. Macrophages stop functioning properly. d. Opportunistic infections and cancer are leading causes of death.

A nurse develops a plan of care for an older client who has a fluid overload. What interventions will the nurse include in this client's care plan? (Select all that apply.) Select one or more: a. Calculate pulse pressure with each blood pressure reading. b. Assess skin turgor using the back of the client's hand. c. Assess for pitting edema in dependent body areas. d. Monitor trends in the client's daily weights. e. Assist the client to change positions frequently. f. Teach client and family how to read food labels for sodium.

a. Calculate pulse pressure with each blood pressure reading. c. Assess for pitting edema in dependent body areas. d. Monitor trends in the client's daily weights. e. Assist the client to change positions frequently. f. Teach client and family how to read food labels for sodium.

A nurse in the oncology clinic is providing preoperative education to a client just diagnosed with cancer. The client has been scheduled for surgery in 3 days. What action by the nurse is best? Select one: a. Call the client at home the next day to review teaching. b. Give the client information about a cancer support group. c. Provide all the preoperative instructions in writing. d. Reassure the client that surgery will be over soon.

a. Call the client at home the next day to review teaching.

The nurse is caring for an older client who had a total knee arthroplasty. Prior to surgery, the client lived alone independently. With which interprofessional health care team members will the nurse collaborate to ensure positive client outcomes? (Select all that apply.) Select one or more: a. Case manager b. Mental health counselor c. Physical therapist d. Occupational therapist e. Speech-language pathologist f. Clergy/Spiritual leader

a. Case manager c. Physical therapist

A nurse is caring for four clients receiving pain medication. After the hand-off report, which client would the nurse see first? a. Client with a Pasero Scale score of 4 b. Client with a verbal pain report of 9 c. Client with a heart rate of 104 beats/min d. Client who is crying and agitated

a. Client with a Pasero Scale score of 4

A nurse on the postoperative inpatient unit receives hand-off report on four clients using patient-controlled analgesia (PCA) pumps. Which client would the nurse see first? a. Client with a respiratory rate of 8 breaths/min. b. Client who appears to be sleeping soundly. c. Client with no bolus request in 6 hours. d. Client who is pressing the button every 10 minutes.

a. Client with a respiratory rate of 8 breaths/min.

A nurse recently hired to the preoperative area learns that certain clients are at higher risk for venous thromboembolism (VTE). Which clients are considered to be at high risk? (Select all that apply.) a. Client with severe heart failure b. 50 years of age or older c. Client who smokes d. Wheelchair-bound client e. Morbidly obese client f. Client with a humerus fracture

a. Client with severe heart failure c. Client who smokes d. Wheelchair-bound client e. Morbidly obese client

The nurse caring for clients who have cancer understands that the general consequences of cancer include which client problems? (Select all that apply.) Select one or more: a. Clotting abnormalities from thrombocythemia b. Increased risk of infection from white blood cell deficits c. Nutritional deficits such as early satiety and cachexia d. Potential for reduced gas exchange e. Various motor and sensory deficits f. Increased risk of bone fractures

a. Clotting abnormalities from thrombocythemia b. Increased risk of infection from white blood cell deficits c. Nutritional deficits such as early satiety and cachexia d. Potential for reduced gas exchange e. Various motor and sensory deficits f. Increased risk of bone fractures

What statements about the complement system are correct? (Select all that apply.) Select one or more: a. Comprised of 20 types of inactive plasma proteins. b. Act as enzymes when activated to enhance innate immunity. c. Phagocytize foreign invaders quickly by destroying their membranes. d. Sticks to the antigen and forms a membrane attack complex. e. Maintain and prolong inflammation from non-self cells. f. Is part of the innate immune system.

a. Comprised of 20 types of inactive plasma proteins. b. Act as enzymes when activated to enhance innate immunity. d. Sticks to the antigen and forms a membrane attack complex. f. Is part of the innate immune system.

A nurse cares for many clients with pressure injuries. What actions by the nurse are considered best practice? (Select all that apply.) Select one or more: a. Conduct ongoing assessments that include pain. b. Use normal saline to cleanse around the pressure injury. c. Soak eschar daily until it softens and can be removed. d. Consult with a registered dietitian nutritionist. e. Use antimicrobial agents to clean wounds that are infected. f. Consider the use of adjuvant therapies for nonhealing wounds.

a. Conduct ongoing assessments that include pain. b. Use normal saline to cleanse around the pressure injury. d. Consult with a registered dietitian nutritionist. e. Use antimicrobial agents to clean wounds that are infected. f. Consider the use of adjuvant therapies for nonhealing wounds.

The nurse providing direct client care uses specific practices to reduce the chance of acquiring infection with human immune deficiency virus (HIV) from clients. Which practice is most effective? Select one: a. Consistent use of Standard Precautions b. Double-gloving before body fluid exposure c. Labeling charts and armbands "HIV+" d. Wearing a mask within 3 feet (1 m) of the client

a. Consistent use of Standard Precautions

A client with HIV-II is hospitalized for an unrelated condition, and several medications are prescribed in addition to the regimen already being used. What action by the nurse is most important? Select one: a. Consult with the pharmacy about drug interactions. b. Ensure that the client understands the new medications. c. Give the new drugs without considering the old ones. d. Schedule all medications at standard times.

a. Consult with the pharmacy about drug interactions.

A client is hospitalized and on multiple antibiotics. The client develops frequent diarrhea. What action by the nurse is most important? Select one: a. Consult with the primary health care provider about obtaining stool cultures. b. Delegate frequent perianal care to assistive personnel. c. Place the client on NPO status until the diarrhea resolves. d. Request a prescription for an antidiarrheal medication.

a. Consult with the primary health care provider about obtaining stool cultures.

A nurse cares for a client who reports discomfort related to eczematous dermatitis. Which nonpharmacologic comfort measures would the nurse implement? (Select all that apply.) Select one or more: a. Cool, moist compresses b. Topical corticosteroids c. Heating pad d. Tepid bath with colloidal oatmeal e. Back rub with baby oil

a. Cool, moist compresses d. Tepid bath with colloidal oatmeal

A nurse is interested in making interprofessional work a high priority. Which actions by the nurse best demonstrate this skill? (Select all that apply.) a. Coordinates discharge planning for home safety. b. Shows the nursing care plans to other disciplines. c. Delegate tasks to unlicensed personnel appropriately. d. Routinely asks other disciplines about client progress. e. Consults with other disciplines on client care.

a. Coordinates discharge planning for home safety. c. Delegate tasks to unlicensed personnel appropriately. d. Routinely asks other disciplines about client progress. e. Consults with other disciplines on client care.

The nurse is caring for a client diagnosed with HIV-II. The client's CD4+ cell count is 399/mm3 (0.399 × 109/L). What action by the nurse is best? Select one: a. Counsel the client on safer sex practices/abstinence. b. Encourage the client to abstain from alcohol. c. Facilitate genetic testing for CD4+ CCR5/CXCR4 co-receptors. d. Help the client plan high-protein/iron meals.

a. Counsel the client on safer sex practices/abstinence.

The nurse working with oncology clients understands that which age-related change increases the older client's susceptibility to infection during chemotherapy? Select one: a. Decreased immune function b. Diminished nutritional stores c. Existing cognitive deficits d. Poor physical reserves

a. Decreased immune function

A nurse is studying pain sources. Which statements accurately describe different types of pain? (Select all that apply.) a. Deep somatic pain is pain arising from bone and connective tissues. b. Somatic pain originates from skin and subcutaneous tissues. c. Nociceptive pain originates from abnormal pain processing. d. Neuropathic pain sometimes accompanies amputation. e. Visceral pain is often diffuse and poorly localized.

a. Deep somatic pain is pain arising from bone and connective tissues. b. Somatic pain originates from skin and subcutaneous tissues. d. Neuropathic pain sometimes accompanies amputation. e. Visceral pain is often diffuse and poorly localized.

A nurse is caring for a client who has the following laboratory results: potassium 2.4 mEq/L (2.4 mmol/L), magnesium 1.8 mEq/L (0.74 mmol/L), calcium 8.5 mEq/L (2.13 mmol/L), and sodium 144 mEq/L (144 mmol/L). Which assessment does the nurse complete first? Select one: a. Depth of respirations b. Bowel sounds c. Grip strength d. Electrocardiography

a. Depth of respirations

A nurse cares for an adult client who has received genetic testing. The patient's mother asks to receive the results of the genetic tests. Which action would the nurse take? a. Direct the mother to speak with the client and support the client's decision to share or not share the results. b. Obtain a signed consent from the client allowing test results to be released to the mother. c. Invite the mother and other family members to participate in genetic counseling with the client.

a. Direct the mother to speak with the client and support the client's decision to share or not share the results.

The nurse is assessing a client for signs and symptoms of systemic lupus erythematosus (SLE). Which of the following would be consistent with this disorder? (Select all that apply.) Select one or more: a. Discoid rash on skin exposed to sunlight b. Urinalysis positive for casts and protein c. Painful, deformed small joints d. Pain on inspiration e. Thrombocytosis f. Serum positive for antinuclear antibodies (ANA)

a. Discoid rash on skin exposed to sunlight b. Urinalysis positive for casts and protein d. Pain on inspiration f. Serum positive for antinuclear antibodies (ANA)

The nurse is caring for a client who has a nasogastric tube (NGT). Which actions would the nurse take for client care? (Select all that apply.) a. Disconnect suction when auscultating bowel peristalsis. b. Flush the tube with water every hour to ensure patency. c. Assess for proper placement of the tube every 4 hours or per agency policy. d. Secure the NG tube to the client's chin. e. Monitor the client's skin around the tube site for irritation.

a. Disconnect suction when auscultating bowel peristalsis. c. Assess for proper placement of the tube every 4 hours or per agency policy. e. Monitor the client's skin around the tube site for irritation.

The nurse is assessing a client for chronic osteomyelitis. Which features distinguish this from the acute form of the disease? (Select all that apply.) Select one or more: a. Draining sinus tracts b. High fevers c. Presence of foot ulcers d. Swelling and redness e. Tenderness or pain

a. Draining sinus tracts c. Presence of foot ulcers

A nurse is planning postoperative care for a client following a total hip arthroplasty. What nursing interventions would help prevent venous thromboembolism for this client? (Select all that apply.) Select one or more: a. Early ambulation b. Fluid restriction c. Quadriceps-setting exercises d. Compression stockings/devices e. Anticoagulant drug therapy

a. Early ambulation c. Quadriceps-setting exercises d. Compression stockings/devices e. Anticoagulant drug therapy

The nurse assesses clients for the cardinal signs of inflammation. Which signs/symptoms does this include? (Select all that apply.) Select one or more: a. Edema b. Pulselessness c. Pallor d. Redness e. Warmth f. Decreased function

a. Edema d. Redness e. Warmth f. Decreased function

The nurse is caring for a client who is diagnosed with a complete small bowel obstruction. For what priority problem is this client most likely at risk? a. Electrolyte imbalance b. Obstipation c. Abdominal distention d. Nausea

a. Electrolyte imbalance

A nurse plans care for a client who is recovering from open reduction and internal fixation (ORIF) surgery for a right hip fracture. Which interventions would the nurse include in this client's plan of care? (Select all that apply.) Select one or more: a. Elevate heels off the bed with a pillow. b. Ambulate the client on the first postoperative day. c. Push the client's patient-controlled analgesia button. d. Re-position the client every 2 hours. e. Use pillows to encourage subluxation of the hip.

a. Elevate heels off the bed with a pillow. b. Ambulate the client on the first postoperative day. d. Re-position the client every 2 hours.

A nurse working in the preoperative holding area performs which functions to ensure client safety? (Select all that apply.) a. Ensure that the client has an armband on. b. Check that consent is on the chart. c. Assess the client for fall risks. d. Have the client help mark the surgical site. e. Allow small sips of plain water. f. Allow the client to use the toilet before giving sedation.

a. Ensure that the client has an armband on. b. Check that consent is on the chart. c. Assess the client for fall risks. d. Have the client help mark the surgical site. f. Allow the client to use the toilet before giving sedation.

A hospitalized client is placed on Contact Precautions. The client needs to have a computed tomography (CT) scan. What action by the nurse is most appropriate? Select one: a. Ensure that the radiology department is aware of the Isolation Precautions. b. Plan to travel with the client to ensure appropriate precautions are used. c. No special precautions are needed when this client leaves the unit. d. Notify the primary health care provider that the client cannot leave the room.

a. Ensure that the radiology department is aware of the Isolation Precautions.

A client hospitalized for chemotherapy has a hemoglobin of 6.1 mg/dL (61 mmol/L). The client is symptomatic but refuses blood transfusions. What medication does the nurse prepare to administer? Select one: a. Epoetin alfa b. Filgrastim c. Mesna d. Dexrazoxane

a. Epoetin alfa

A client who has rheumatoid arthritis is prescribed etanercept. What health teaching by the nurse about this drug is appropriate? Select one: a. Giving subcutaneous injections b. Having a chest x-ray once a year c. Taking the medication with food d. Using heat on the injection site

a. Giving subcutaneous injections

A client is getting out of bed into the chair for the first time after an uncemented total hip arthroplasty. What action by the nurse is appropriate? Select one: a. Have adequate help to transfer the patient. b. Provide socks so the patient can slide easier. c. Tell the patient full weight bearing is allowed. d. Use a footstool to elevate the patient's leg.

a. Have adequate help to transfer the patient.

The nurse is teaching an elderly client the risks of infection for older adults. Which of the following factors would the nurse include in the education? (Select all that apply.) Select one or more: a. Higher risk for respiratory tract and genitourinary infections. b. May not have a fever with severe infection. c. Show expected changes in white blood cell counts. d. Should receive influenza, pneumococcal, and shingles vaccinations. e. Skin tests for tuberculosis may be falsely negative. f. Booster vaccinations are not likely needed as one ages.

a. Higher risk for respiratory tract and genitourinary infections. b. May not have a fever with severe infection. d. Should receive influenza, pneumococcal, and shingles vaccinations. e. Skin tests for tuberculosis may be falsely negative.

The nurse is teaching an elderly client the risks of infection for older adults. Which of the following factors would the nurse include in the education? (Select all that apply.) Select one or more: a. Higher risk for respiratory tract and genitourinary infections. b. May not have a fever with severe infection. c. Show expected changes in white blood cell counts. d. Should receive influenza, pneumococcal, and shingles vaccinations. e. Skin tests for tuberculosis may be falsely negative. f. Booster vaccinations are not likely needed as one ages.

a. Higher risk for respiratory tract and genitourinary infections. b. May not have a fever with severe infection. d. Should receive influenza, pneumococcal, and shingles vaccinations. e. Skin tests for tuberculosis may be falsely negative.

A nurse is caring for clients with electrolyte imbalances on a medical-surgical unit. Which clinical signs and symptoms are correctly paired with the contributing electrolyte imbalance? (Select all that apply.) Select one or more: a. Hypokalemia—muscle weakness with respiratory depression b. Hypermagnesemia—bradycardia and hypotension c. Hyponatremia—decreased level of consciousness d. Hypercalcemia—positive Trousseau and Chvostek signs e. Hypomagnesemia—hyperactive deep tendon reflexes f. Hypernatremia—weak peripheral pulses

a. Hypokalemia—muscle weakness with respiratory depression b. Hypermagnesemia—bradycardia and hypotension c. Hyponatremia—decreased level of consciousness e. Hypomagnesemia—hyperactive deep tendon reflexes f. Hypernatremia—weak peripheral pulses

The nurse is learning about immunoglobulins. . Which principles does the nurse learn? (Select all that apply.) Select one or more: a. IgA is found in high concentrations in secretions from mucous membranes. b. IgD is present in the highest concentrations in mucous membranes. c. IgE is associated with antibody-mediated hypersensitivity reactions. d. IgG comprises the majority of the circulating antibody population. e. IgM is the first antibody formed by a newly sensitized B-cell.

a. IgA is found in high concentrations in secretions from mucous membranes. c. IgE is associated with antibody-mediated hypersensitivity reactions. d. IgG comprises the majority of the circulating antibody population. e. IgM is the first antibody formed by a newly sensitized B-cell.

A nurse is studying the function of immunoglobulins. Which immunoglobulins are correctly matched to their function? (Select all that apply.) Select one or more: a. IgA: most responsible for preventing infection in the respiratory tracts, the GI tract, and the genitourinary tract. b. IgD: provides protection against parasite infestations, especially helminths. c. IgE: associated with antibody-mediated immediate hypersensitivity reactions. d. IgG: activates classic complement pathway and enhances neutrophil and macrophage actions. e. IgM: first antibody formed by a newly sensitized B-lymphocyte plasma cell.

a. IgA: most responsible for preventing infection in the respiratory tracts, the GI tract, and the genitourinary tract. c. IgE: associated with antibody-mediated immediate hypersensitivity reactions. d. IgG: activates classic complement pathway and enhances neutrophil and macrophage actions. e. IgM: first antibody formed by a newly sensitized B-lymphocyte plasma cell.

The nurse is caring for a client who recently sustained a sports injury to his right leg. What nursing interventions are appropriate for this client? (Select all that apply.) Select one or more: a. Immobilize the right leg. b. Apply heat immediately after the injury. c. Use compression to support the leg. d. Obtain an x-ray to detect possible fracture. e. Elevate the right leg to decrease swelling. f. Administer an opioid every 4 to 6 hours.

a. Immobilize the right leg. c. Use compression to support the leg. d. Obtain an x-ray to detect possible fracture. e. Elevate the right leg to decrease swelling.

A nurse assesses a client who is admitted for treatment of fluid overload. Which signs and symptoms does the nurse expect to find? (Select all that apply.) Select one or more: a. Increased pulse rate b. Distended neck veins c. Decreased blood pressure d. Warm and pink skin e. Skeletal muscle weakness f. Visual disturbances

a. Increased pulse rate b. Distended neck veins e. Skeletal muscle weakness f. Visual disturbances

A nurse is working with a client who has a painful rash consisting of grouped weeping and crusting lesions in distinct lines. What actions by the nurse are most appropriate? (Select all that apply.) Select one or more: a. Instruct the client to report lesions near the eyes. b. Have the client take long, hot baths to soak the lesions. c. Show the client how to make a baking soda compress. d. Advise the client to avoid exposure to UV light rays. e. Demonstrate proper use of antifungal medications. f. Review appropriate hygiene measures.

a. Instruct the client to report lesions near the eyes. c. Show the client how to make a baking soda compress. d. Advise the client to avoid exposure to UV light rays.

A nurse teaches a client with a fractured tibia about external fixation. Which advantages of external fixation for the immobilization of fractures would the nurse share with the client? (Select all that apply.) Select one or more: a. It leads to minimal blood loss. b. It allows for early ambulation. c. It decreases the risk of infection. d. It increases blood supply to tissues. e. It promotes healing.

a. It leads to minimal blood loss. b. It allows for early ambulation. e. It promotes healing.

The nurse caring for clients admitted for infectious diseases understands what information about emerging global diseases and bioterrorism? Select one: a. Many infections are or could be spread by international travel. b. Safer food preparation practices have decreased foodborne illnesses. c. The majority of Americans have adequate innate immunity to smallpox. d. Plague produces a mild illness and generally has a low mortality rate.

a. Many infections are or could be spread by international travel.

A nurse is studying the functions of specific leukocytes. Which leukocytes are matched correctly with their function? (Select all that apply.) Select one or more: a. Monocyte: matures into a macrophage. b. Basophil: releases vasoactive amines during an allergic reaction. c. Plasma cell: secretes immunoglobulins in response to the presence of a specific antigen. d. Cytotoxic T-cells: attacks and destroys ingested poisons and toxins. e. Natural killer cell: nonselectively attacks non-self cells. f. Regulator T-cells: become sensitized for self-recognition in the bone marrow.

a. Monocyte: matures into a macrophage. c. Plasma cell: secretes immunoglobulins in response to the presence of a specific antigen. e. Natural killer cell: nonselectively attacks non-self cells.

According to the WHO, what does primary care involve? (Select all that apply.) a. Multisectoral policy and action b. Empowered people and communities c. Essential public functions d. Elimination of chronic diseases. e. Primary care f. Priority consideration of chronic diseases

a. Multisectoral policy and action b. Empowered people and communities e. Primary care

A client reports a great deal of pain following a fairly minor operation. The surgeon leaves a prescription for the nurse to administer a placebo instead of pain medication. Which actions by the nurse are most appropriate? (Select all that apply.) a. Notify the nurse manager of the placebo prescription. b. Delegate administration of the placebo to another nurse. c. Consult with the surgeon and voice objections. d. Tell the client what medications were prescribed. e. Give the placebo and reassess the client's pain.

a. Notify the nurse manager of the placebo prescription. c. Consult with the surgeon and voice objections.

The nurse utilizing evidence-based practice (EBP) considers which factors when planning care? (Select all that apply.) a. Nurse's expertise b. Client preferences c. Values of the client d. Cost-saving measures e. Plan-do-study-act model f. Research findings

a. Nurse's expertise b. Client preferences c. Values of the client f. Research findings

A client is being administered the first dose of belimumab for a systemic lupus erythematosus flare. What actions by the nurse are most appropriate? (Select all that apply.) Select one or more: a. Observe the client for at least 2 hours afterward. b. Instruct the client about the monthly infusion schedule. c. Inform the client not to drive or sign legal papers for 24 hours. d. Ensure emergency equipment is working and nearby. e. Make a follow-up appointment for a lipid panel in 2 months. f. Instruct the client to hold other medications for 72 hours.

a. Observe the client for at least 2 hours afterward. d. Ensure emergency equipment is working and nearby.

A hospitalized older adult has been assessed at high risk for skin breakdown. Which actions does the registered nurse (RN) delegate to the assistive personnel (AP)? (Select all that apply.) a. Obtain a pressure-relieving mattress. b. Turn the client every 2 hours. c. Keep the client's skin dry. d. Document Braden Scale results. e. Assess skin redness when turning.

a. Obtain a pressure-relieving mattress. b. Turn the client every 2 hours. c. Keep the client's skin dry.

A rehabilitation nurse assesses a client upon admission. Which assessments would the nurse complete to determine actual or potential interruption in skin and tissue integrity? (Select all that apply.) a. Oxygen saturation b. Nutrition c. Urinary output d. Cognitive abilities e. Functional mobility f. Spiritual needs

a. Oxygen saturation b. Nutrition c. Urinary output d. Cognitive abilities e. Functional mobility

A nurse cares for an older adult who is admitted to the hospital with complications of diverticulitis. Which actions would the nurse include in the client's plan of care? (Select all that apply.) a. Palpate the abdomen for distention. b. Provide the client with a high-fiber diet. c. Administer pain medications as prescribed. d. Evaluate stools for occult blood. e. Assess for sudden changes in mental status.

a. Palpate the abdomen for distention. c. Administer pain medications as prescribed. d. Evaluate stools for occult blood. e. Assess for sudden changes in mental status.

A nurse on the postoperative nursing unit provides care to reduce the incidence of surgical wound infection. What actions are best to achieve this goal? (Select all that apply.) a. Performing proper hand hygiene b. Removing and replacing wet dressings c. Administering antibiotics for 72 hours d. Disposing of dressings properly

a. Performing proper hand hygiene b. Removing and replacing wet dressings d. Disposing of dressings properly

A nurse orienting to the postoperative area learns which principles about the postoperative period? (Select all that apply.) a. Phase I care may last for several days in some clients. b. Some clients may be discharged directly after phase I. c. Phase II ends when the client is at a pre-surgical level of consciousness and baseline oxygen saturation, and vital signs are stable. d. Phase I requires intensive care unit monitoring. e. All phases require the client to be in the hospital. f. Vital signs may be taken only once a day in phase III.

a. Phase I care may last for several days in some clients. b. Some clients may be discharged directly after phase I. c. Phase II ends when the client is at a pre-surgical level of consciousness and baseline oxygen saturation, and vital signs are stable. f. Vital signs may be taken only once a day in phase III.

A nurse assesses a client who has a peripherally inserted central catheter (PICC). For which common complications will the nurse assess? (Select all that apply.) Select one or more: a. Phlebitis b. Pneumothorax c. Thrombophlebitis d. Excessive bleeding e. Extravasation f. Pneumothorax g. Infiltration h. catheter-related bloodstream infection

a. Phlebitis c. Thrombophlebitis h. catheter-related bloodstream infection

The nurse is caring for a client who has perineal surgical wound. Which actions would the nurse take to promote comfort and wound healing? (Select all that apply.) a. Place an absorbent dressing over the wound. b. Apply warm compresses three to four times a day. c. Use a rubber donut device when sitting up. d. Instruct the client to wear boxer shorts. e. Assist the client into a side-lying position

a. Place an absorbent dressing over the wound. b. Apply warm compresses three to four times a day. e. Assist the client into a side-lying position

A client who had a surgical fractured femur repair reports new-onset shortness of breath and increased respirations. What is the nurse's first action? Select one: a. Place the client in a high-Fowler position. b. Document the client's oxygen saturation level. c. Start oxygen therapy at 2 L/min via nasal cannula. d. Contact the primary health care provider.

a. Place the client in a high-Fowler position.

A preoperative nurse is reviewing morning laboratory values on four clients waiting for surgery. Which result warrants immediate communication with the surgical team? a. Potassium: 2.9 mEq/L (2.9 mmol/L) b. Sodium: 134 mEq/L (134 mmol/L) c. Hemoglobin: 14.8 mg/dL (148 mmol/L) d. Creatinine: 1.2 mg/dL (106.1 umol/L)

a. Potassium: 2.9 mEq/L (2.9 mmol/L)

A nurse is caring for a client who has methicillin-resistant Staphylococcus aureus(MRSA) infection cultured from the urine. What action by the nurse is most appropriate? Select one: a. Prepare to administer vancomycin. b. Strictly limit visitors to immediate family only. c. Wash hands only after taking off gloves after care. d. Wear a respirator when handling urine output.

a. Prepare to administer vancomycin.

Once the nurse has considered all possible collaborative and client problems, what action does the nurse take next? a. Prioritize the hypotheses. b. Act on the observed cues. c. Generate solutions. d. Determine desired outcomes.

a. Prioritize the hypotheses.

An older adult client takes medication three times a day and becomes confused about which medication should be taken at which time. The client refuses to use a pill sorter with slots for different times, saying "Those are for old people." What action by the nurse would be most helpful? a. Put color-coded stickers on the bottle caps. b. Arrange medications by time in a drawer. c. Encourage the client to use easy-open tops. d. Write a list of when to take each medication.

a. Put color-coded stickers on the bottle caps.

A nurse is assessing a client who has an electrolyte imbalance related to renal failure. For which potential complications of this electrolyte imbalance does the nurse assess? (Select all that apply.) Select one or more: a. Reports of palpitations b. Slow, shallow respirations c. Orthostatic hypotension d. Paralytic ileus e. Skeletal muscle weakness f. Tall, peaked T waves on ECG

a. Reports of palpitations e. Skeletal muscle weakness f. Tall, peaked T waves on ECG

A nurse assesses an older client who is scratching and rubbing white ridges on the skin between the fingers and on the wrists. Which action would the nurse take? Select one: a. Request a prescription for permethrin. b. Administer an antihistamine. c. Assess the client's airway. d. Apply gloves to minimize friction.

a. Request a prescription for permethrin.

A nurse is caring for a client admitted for Non-Hodgkin's lymphoma and chemotherapy. The client reports nausea, flank pain, and muscle cramps. What action by the nurse is most important? Select one: a. Request an order for serum electrolytes and uric acid. b. Increase the client's IV infusion rate. c. Instruct assistive personnel to strain all urine. d. Administer an IV antiemetic.

a. Request an order for serum electrolytes and uric acid.

A nurse manager wishes to ensure that the nurses on the unit are practicing at their highest levels of competency. Which areas would the manager assess to determine if the nursing staff demonstrate competency according to the Institute of Medicine (IOM) report Health Professions Education: A Bridge to Quality? (Select all that apply.) a. Routinely using informatics in practice. b. Providing family-focused care. c. Collaborating with an interprofessional team. d. Implementing evidence-based care. e. Formalizing systems thinking when implementing care f. Using quality improvement in client care

a. Routinely using informatics in practice c. Collaborating with an interprofessional team d. Implementing evidence-based care f. Using quality improvement in client care

The nurse caring for clients assesses their daily laboratory profiles. Which lab results are considered to be in the normal range? (Select all that apply.) Select one or more: a. Segmented neutrophils: 68% b. Bands: 19% c. Monocytes: 12% d. Lymphocytes: 38% e. Eosinophils: 2% f. Basophils: 1%

a. Segmented neutrophils: 68% d. Lymphocytes: 38% e. Eosinophils: 2% f. Basophils: 1%

The nurse manager is conducting an annual evaluation of a staff nurse and is appraising the nurse's clinical reasoning. What nurse actions does the manager observe to help form this judgment? (Select all that apply.) a. Setting priorities. b. Anticipating consequences of actions. c. Noticing cues. d. Delegating appropriately. e. Interpreting data

a. Setting priorities b. Anticipating consequences of actions c. Noticing cues e. Interpreting data

A nurse learns that there are physical consequences to unrelieved pain. Which factors are included in this problem? (Select all that apply.) a. Slower healing b. Decreased immune response c. Increased gastrointestinal (GI) motility d. Possible immobility e. Negative quality of life f. Development of chronic pain

a. Slower healing b. Decreased immune response d. Possible immobility e. Negative quality of life f. Development of chronic pain

A nurse is assessing clients with pressure injuries. Which wound description is correctly matched to its description? Select one: a. Suspected deep tissue injury: nonblanchable deep purple or maroon. b. Stage 2: may have visible adipose tissue and slough. c. Stage 3: may have a pink or red wound bed. d. Stage 4: wound bed is obscured with eschar or slough.

a. Suspected deep tissue injury: nonblanchable deep purple or maroon.

The nurse learns that which age-related changes increase the potential for complications of burns? (Select all that apply.) Select one or more: a. Thinner skin b. Slower healing time c. Decreased mobility d. Hyperresponsive immune response e. Increased risk of unnoticed sepsis f. Pre-existing conditions

a. Thinner skin b. Slower healing time c. Decreased mobility e. Increased risk of unnoticed sepsis f. Pre-existing conditions

What does the nurse learn about the function of colony-stimulating factor? Select one: a. Triggers the bone marrow to shorten the time needed to produce mature WBCs. b. Causes capillary leak in acute inflammation. c. Responsible for creating exudate (pus) at infectious sites. d. Dilates blood vessels at the site of inflammation leading to hyperemia.

a. Triggers the bone marrow to shorten the time needed to produce mature WBCs.

The nurse is studying hypersensitivity reactions. Which reactions are correctly matched with their hypersensitivity types? (Select all that apply.) Select one or more: a. Type I—examples include hay fever and anaphylaxis. b. Type II—mediated by action of immunoglobulin M (IgM). c. Type III—immune complex deposits in blood vessel walls. d. Type IV—examples are poison ivy and transplant rejection. e. Type IV—involve both antibodies and complement.

a. Type I—examples include hay fever and anaphylaxis. c. Type III—immune complex deposits in blood vessel walls. d. Type IV—examples are poison ivy and transplant rejection.

A nurse is preparing to give a client ketorolac intravenously for pain. Which assessment findings would lead the nurse to consult with the primary health care provider? a. Urine output of 20 mL/2 hr b. Bilateral lung crackles c. Hypoactive bowel sounds d. Self-reported pain of 3/10

a. Urine output of 20 mL/2 hr

A nurse on the postoperative unit administers many opioid analgesics. Which actions by the nurse are best to prevent unwanted sedation as a complication of these medications? (Select all that apply.) a. Use an oximeter to monitor clients receiving analgesia. b. Identify clients at high risk for unwanted sedation. c. Avoid using other medications that cause sedation. d. Give the lowest dose that produces good control. e. Delay giving medication if the client is sleeping.

a. Use an oximeter to monitor clients receiving analgesia. b. Identify clients at high risk for unwanted sedation. c. Avoid using other medications that cause sedation. d. Give the lowest dose that produces good control.

A preoperative nurse is assessing a client prior to surgery. Which information would be most important for the nurse to relay to the surgical team? a. Use of multiple herbs and supplements b. History of lactose intolerance c. Allergy to bee and wasp stings d. No previous experience with surgery

a. Use of multiple herbs and supplements

A rehabilitation nurse prepares to move a client who has new bilateral leg amputations. Which is the best approach? a. Utilize the facility's mechanical lift to move the client. b. Ask several members of the health care team to carry the client. c. Use the bear-hug method to transfer the client safely. d. Consult physical therapy before performing all transfers.

a. Utilize the facility's mechanical lift to move the client.

A nurse begins a job at a Veterans Administration Hospital and asks why so much emphasis is on HIV testing for the veterans. What reasons is this nurse given? (Select all that apply.) Select one or more: a. Veterans have a high prevalence of substance abuse. b. Many veterans may engage in high risk behaviors. c. Many older veterans may not know their risks. d. Everyone should know their HIV status. e. Belief that the VA has tested them and would notify them if positive.

a. Veterans have a high prevalence of substance abuse. b. Many veterans may engage in high risk behaviors. c. Many older veterans may not know their risks. d. Everyone should know their HIV status. e. Belief that the VA has tested them and would notify them if positive.

A nurse working with older adults assesses them for common potential adverse medication effects. For what does the nurse assess? (Select all that apply.) a. Weakness b. Mania c. Constipation d. Dehydration e. Anorexia f. Urinary incontinence

a. Weakness c. Constipation d. Dehydration e. Anorexia

A nurse cares for several clients on an inpatient unit. Which infection control measures will the nurse implement? (Select all that apply.) Select one or more: a. Wear a gown when contact of clothing with body fluids is anticipated. b. Teach clients and visitors respiratory hygiene techniques. c. Obtain powered air purifying respirators for all staff members. d. Do not use alcohol-based hand rub between client contacts. e. Disinfect frequently touched surfaces in client-care areas.

a. Wear a gown when contact of clothing with body fluids is anticipated. b. Teach clients and visitors respiratory hygiene techniques. e. Disinfect frequently touched surfaces in client-care areas.

A postanesthesia care unit (PACU) nurse is assessing a postoperative client with a nasogastric (NG) tube. What laboratory values would warrant intervention by the nurse? (Select all that apply.) a. pH: 7.68 b. Hemoglobin: 7.8 mg/dL (78 mmol/L) c. Blood glucose: 120 mg/dL (6.7 mmol/L) d. Potassium: 2.9 mEq/L (2.9 mmol/L) e. Sodium: 142 mEq/L (142 mmol/L)

a. pH: 7.68 b. Hemoglobin: 7.8 mg/dL (78 mmol/L) d. Potassium: 2.9 mEq/L (2.9 mmol/L)

A nurse teaches a client who is at risk for colorectal cancer. Which dietary recommendation would the nurse teach the client? a. "Foods high in animal fat help to protect the intestinal mucosa." b. "Add vegetables such as broccoli and cauliflower to your diet." c. "Eat low-fiber and low-residual foods." d. "White rice and bread are easier to digest."

b. "Add vegetables such as broccoli and cauliflower to your diet."

A client has been newly diagnosed with systemic lupus erythematosus and is reviewing self-care measures with the nurse. Which statement by the client indicates a need to review the material? Select one: a. "I will avoid direct sunlight as much as possible." b. "Baby powder is good for the constant sweating." c. "Grouping errands will help prevent fatigue." d. "Rest time will have to become a priority."

b. "Baby powder is good for the constant sweating."

A client tells the oncology nurse about an upcoming vacation to the beach to celebrate completing radiation treatments for cancer. What response by the nurse is mostappropriate? Select one: a. "Avoid getting salt water on the radiation site." b. "Do not expose the radiation area to direct sunlight." c. "Have a wonderful time and enjoy your vacation!" d. "Remember you should not drink alcohol for a year."

b. "Do not expose the radiation area to direct sunlight."

A nurse consults a genetic counselor for a client whose mother has Huntington disease and is considering genetic testing. The client states, "I know I want this test. Why do I need to see a counselor?" How would the nurse respond? a. "The counselor will advise you on whether you can have children or need to adopt." b. "Genetic testing can be a stressful experience. Counseling can provide support and education throughout the process." c. "Genetic testing is expensive. The counselor will advocate for you and help you obtain financial support." d. "There is no cure for this disease. The counselor will determine if there is any benefit to genetic testing."

b. "Genetic testing can be a stressful experience. Counseling can provide support and education throughout the process."

The nurse assesses a client with diabetes and osteoarthritis (OA) during a checkup. The nurse notes the client's blood glucose readings have been elevated. What question by the nurse is most appropriate? Select one: a. "Are you following the prescribed diabetic diet?" b. "Have you been taking glucosamine supplements?" c. "How much exercise do you really get each week?" d. "You're still taking your diabetic medication, right?"

b. "Have you been taking glucosamine supplements?"

A nurse assesses a client who presents with an increase in psoriatic lesions. Which questions would the nurse ask to identify a possible trigger for worsening of this client's psoriatic lesions? (Select all that apply.) Select one or more: a. "Have you eaten a large amount of chocolate lately?" b. "Have you been under a lot of stress lately?" c. "Have you recently used a public shower?" d. "Have you been out of the country recently?" e. "Have you recently had any other health problems?" f. "Have you changed any medications recently?"

b. "Have you been under a lot of stress lately?" e. "Have you recently had any other health problems?" f. "Have you changed any medications recently?"

A nurse is teaching a client who has itchy, raised red patches covered with a silvery white scale how to care for this disorder. What statement by the client shows a need for further information? Select one: a. "At the next family reunion, I'm going to ask my relatives if they have anything similar." b. "I have to make sure I keep my lesions covered, so I do not spread this to others." c. "I must avoid large crowds and sick people while I am taking adalimumab." d. "I will buy a good quality emollient to put on my skin each day."

b. "I have to make sure I keep my lesions covered, so I do not spread this to others."

After teaching a client who is prescribed adalimumab for severe ulcerative colitis (UC), the nurse assesses the client's understanding. Which statement made by the client indicates a need for further teaching? a. "Nausea and vomiting are common side effects of this drug." b. "I must take this medication with my breakfast each morning." c. "I will avoid large crowds and people who are sick." d. "I should wash my hands after I play with my dog."

b. "I must take this medication with my breakfast each morning."

A nurse cares for a client with colorectal cancer who has a new colostomy. The client states, "I think it would be helpful to talk with someone who has had a similar experience." How would the nurse respond? a. "The ostomy nurse will be able to answer all of your questions." b. "I will make a referral to the United Ostomy Associations of America." c. "I have a good friend with a colostomy who would be willing to talk with you." d. "You'll find that most people with colostomies don't want to talk about them."

b. "I will make a referral to the United Ostomy Associations of America."

After teaching a client who is being treated for dehydration, a nurse assesses the client's understanding. Which statement indicates that the client correctly understoodthe teaching? Select one: a. "I must drink a quart (liter) of water or other liquid each day." b. "I will weigh myself each morning before I eat or drink." c. "I will use a salt substitute when making and eating my meals." d. "I will not drink liquids after 6 p.m. so I won't have to get up at night."

b. "I will weigh myself each morning before I eat or drink."

A client in the oncology clinic reports her family is frustrated at her ongoing fatigue 4 months after radiation therapy for breast cancer. What response by the nurse is mostappropriate? Select one: a. "Are you getting adequate rest and sleep each day?" b. "It is normal to be fatigued even for months afterward." c. "This is not normal and I'll let the primary health care provider know." d. "Try adding more vitamins B and C to your diet."

b. "It is normal to be fatigued even for months afterward."

A client has been prescribed denosumab. What health teaching about this drug is most appropriate for the nurse to include? Select one: a. "Drink at least 8 ounces (240 mL) of water with it." b. "Make appointments to come get your injection." c. "Sit upright for 30 to 60 minutes after taking it." d. "Take the drug on an empty stomach."

b. "Make appointments to come get your injection."

A nurse cares for a client with a new ileostomy. The client states, "I don't think my friends will accept me with this ostomy." How would the nurse respond? a. "A therapist can help you resolve your concerns." b. "Tell me more about your concerns." c. "Your friends will be happy that you are alive." d. "With time you will accept your new body."

b. "Tell me more about your concerns."

A nurse is educating a client about genetic screening. The client asks why red-green color blindness, an X-linked recessive disorder noted in some family members, is expressed more frequently in males than females. How would the nurse respond? a. "Females have two X chromosomes and one is always inactive. This inactivity decreases the effect of the gene." b. "The incidence of X-linked recessive disorders is higher in males because they do not have a second X chromosome to balance expression of the gene." c. "Females have a decreased penetrance rate for this gene mutation and are therefore less likely to express the trait." d. "Males have only one X chromosome, which allows the X-linked recessive disorder to be transmitted from father to son."

b. "The incidence of X-linked recessive disorders is higher in males because they do not have a second X chromosome to balance expression of the gene."

The nurse is teaching a client with mild rheumatoid arthritis (RA) about how to protect synovial joints. Which health teaching will the nurse include? (Select all that apply.) Select one or more: a. "Use small joints rather than larger ones during tasks." b. "Use both hands instead of one with holding objects." c. "When getting out of bed or a chair, use the palms of your hands." d. "Bend your knees instead of your waist and keep your back straight." e. "Do not use multiple pillows under your head to prevent neck flexion." f. "Use a device or rubber grip to open jars or bottle tops." g. "Use long-handled devices such as a hairbrush with an extended handle."

b. "Use both hands instead of one with holding objects." c. "When getting out of bed or a chair, use the palms of your hands." d. "Bend your knees instead of your waist and keep your back straight." e. "Do not use multiple pillows under your head to prevent neck flexion." f. "Use a device or rubber grip to open jars or bottle tops." g. "Use long-handled devices such as a hairbrush with an extended handle."

The nurse teaches a community group ways to prevent Escherichia coli infection. Which statements would the nurse include in this group's teaching? (Select all that apply.) a. "It is not necessary to buy a meat thermometer." b. "Wash your hands after any contact with animals." c. "Avoid swimming in backyard pools and using hot tubs." d. "Stay away from people who are ill with diarrhea." e. "Use separate cutting boards for meat and vegetables."

b. "Wash your hands after any contact with animals." e. "Use separate cutting boards for meat and vegetables."

A nurse cares for a client who recently completed genetic testing and received a negative result. The client states, "I feel guilty because so many of my family members are carriers of this disease and I am not." How would the nurse respond? a. "You are not genetically predisposed for this disease but you could still become ill. Let's discuss a plan for prevention." b. "We usually encourage clients to participate in counseling after receiving test results. Can I arrange this for you?" c. "It is normal to feel this way. I think you would share this news with your family so that they can support you." d. "Since many of your family members are carriers, you would undergo further testing to verify the results are accurate."

b. "We usually encourage clients to participate in counseling after receiving test results. Can I arrange this for you?"

The nurse is assessing a client's pain and has elicited information on the location, quality, intensity, effect on functioning, aggravating and relieving factors, and onset and duration. Which question by the nurse would be best to ask the client for completing a comprehensive pain assessment? a. "Do you attach any spiritual meaning to pain?" b. "What pain rating would be acceptable to you?" c. "Are you worried about addiction to pain pills?" d. "How high would you say your pain tolerance is?"

b. "What pain rating would be acceptable to you?"

The older client's adult child questions the nurse as to why the client is at higher risk for infection when the client's white cell count is within the normal range. What response by the nurse is best? Select one: a. "The white cell count does not tell us everything about immunity." b. "White blood cells are less active in older people so they are not as efficient." c. "Older people typically have poor nutrition which makes them prone to infection." d. "As one ages, immunoglobulins cease to be produced in response to illness."

b. "White blood cells are less active in older people so they are not as efficient."

A client is preparing to have a laparoscopic restorative proctocolectomy with ileo pouch-anal anastomosis (RCA-IPAA). Which preoperative health teaching would the nurse include? a. "You will have to wear an appliance for your permanent ileostomy." b. "You should be able to have better bowel continence after healing occurs." c. "This procedure can be performed under general or regional anesthesia." d. "You will have a large abdominal incision that will require irrigation."

b. "You should be able to have better bowel continence after healing occurs."

A rehabilitation nurse in a skilled nursing facility (SNF) cares for a client who has generalized weakness and needs assistance with activities of daily living. Which exercise would the nurse implement? a. Passive range of motion b. Active range of motion c. Resistive range of motion d. Aerobic exercise

b. Active range of motion

The nurse is caring for a client who just had a kyphoplasty. What nursing care is needed for the client at this time? (Select all that apply.) Select one or more: a. Place the client in a prone position to prevent pressure on the surgical area. b. Apply an ice pack to the surgical area to help relieve pain. c. Assess the client's pain level to compare it with pain before the procedure. d. Take vital signs, including oxygen saturation, frequently. e. Monitor for complications such as bleeding or shortness of breath. f. Perform frequent neurologic assessments and report major changes.

b. Apply an ice pack to the surgical area to help relieve pain. c. Assess the client's pain level to compare it with pain before the procedure. d. Take vital signs, including oxygen saturation, frequently. e. Monitor for complications such as bleeding or shortness of breath. f. Perform frequent neurologic assessments and report major changes.

A client with osteoporosis is going home where the client lives alone. What action by the nurse is best? Select one: a. Refer the client to Meals on Wheels. b. Arrange a home safety evaluation. c. Ensure that the client has a walker at home. d. Help the client look into assisted living.

b. Arrange a home safety evaluation.

Which action by the nurse working with a client best demonstrates respect for autonomy? Autonomy is self-determination. The client would make decisions regarding care. When the nurse obtains a signature on the consent form, assessing if the client still has questions is vital, because without full information the client cannot practice autonomy. Giving accurate information is practicing with veracity. Keeping promises is upholding fidelity. Treating the client fairly is providing social justice. a. Keeps the promises made to the client and family. b. Asks if the client has questions before signing a consent. c. Gives the client accurate information when questioned.

b. Asks if the client has questions before signing a consent.

A client is admitted to the emergency department with a fractured femur resulting from a motor vehicle crash. What the nurse's priority action? Select one: a. Keep the client warm and comfortable. b. Assess airway, breathing, and circulation. c. Maintain the client in a supine position. d. Immobilize the injured extremity with a splint.

b. Assess airway, breathing, and circulation.

A nurse is caring for an older client who exhibits dehydration-induced confusion. Which intervention by the nurse is best? Select one: a. Measure intake and output every 4 hours. b. Assess client further for fall risk. c. Increase the IV flow rate to 250 mL/hr. d. Place the client in a high-Fowler position.

b. Assess client further for fall risk.

A nurse wishes to provide client-centered care in all interactions. Which action by the nurse best demonstrates this concept? a. Thoroughly orients the client and family to the room. b. Assess for cultural influences affecting health care. c. Tells the client and family about all upcoming tests. d. Ensures that all client's basic needs are met.

b. Assess for cultural influences affecting health care.

A visiting nurse is in the home of an older adult and notes a 7-lb weight loss since last month's visit. What actions would the nurse perform first? (Select all that apply.) a. Encourage the client to continue the current exercise plan. b. Assess the client's ability to drive or transportation alternatives. c. Have the client complete a 3-day diet recall diary. d. Teach the client about proper nutrition in the older population. e. Determine if the client has dentures that fit appropriately.

b. Assess the client's ability to drive or transportation alternatives. c. Have the client complete a 3-day diet recall diary. e. Determine if the client has dentures that fit appropriately.

The nurse is caring for a client who has fluid overload. What action by the nurse takes priority? Select one: a. Administer high-ceiling (loop) diuretics. b. Assess the client's lung sounds every 2 hours. c. Place a pressure-relieving overlay on the mattress. d. Weigh the client daily at the same time on the same scale.

b. Assess the client's lung sounds every 2 hours.

A client with HIV-III has been hospitalized with suspected cryptosporidiosis. What physical assessment would be most important with this condition? Select one: a. Auscultating the lungs b. Assessing mucous membranes c. Listening to bowel sounds d. Performing a neurologic examination

b. Assessing mucous membranes

A client with HIV-III has oral thrush and difficulty eating. What actions does the nurse delegate to the assistive personnel (AP)? (Select all that apply.) Select one or more: a. Apply oral anesthetic gels before meals. b. Assist the client with oral care every 2 hours. c. Offer the client frequent sips of cool drinks. d. Provide the client with alcohol-based mouthwash. e. Remind the client to use only a soft toothbrush. f. Offer the client soft foods like gelatin or pudding.

b. Assist the client with oral care every 2 hours. c. Offer the client frequent sips of cool drinks. e. Remind the client to use only a soft toothbrush. f. Offer the client soft foods like gelatin or pudding.

A client with HIV-III is in the hospital with severe diarrhea. What actions does the nurse delegate to assistive personnel (AP)? (Select all that apply.) Select one or more: a. Assessing the client's fluid and electrolyte status b. Assisting the client to get out of bed to prevent falls c. Obtaining a bedside commode if the client is weak d. Providing gentle perianal cleansing after stools e. Reporting any perianal abnormalities

b. Assisting the client to get out of bed to prevent falls c. Obtaining a bedside commode if the client is weak d. Providing gentle perianal cleansing after stools e. Reporting any perianal abnormalities

A nurse manager wants to improve hand-off communication among the staff. What actions by the manager would best help achieve this goal? (Select all that apply.) a. Give raises based on compliance with reporting. b. Attend hand-off rounds to coach and mentor. c. Create a template of suggested topics to include in report. d. Provide education on the SBAR method of communication. e. Encourage staff to ask questions during hand-off.

b. Attend hand-off rounds to coach and mentor. d. Provide education on the SBAR method of communication. e. Encourage staff to ask questions during hand-off.

After teaching a patient with diverticular disease, a nurse assesses the client's understanding. Which menu selection indicates the client correctly understood the teaching? a. Garden salad with a cup of bean soup and a glass of low-fat milk b. Baked fish with steamed carrots and a glass of apple juice c. Spaghetti with meat sauce, a fresh fruit cup, and hot tea d. Roasted chicken with rice pilaf and a cup of coffee with cream

b. Baked fish with steamed carrots and a glass of apple juice

The expert nurse understands that critical thinking requires which elements to be present? (Select all that apply.) a. Grounded in a specific theory. b. Based on logic, creativity, and intuition. c. Requires forming options about evidence. d. Guided by standards. e. Focused on safety and quality. f. Driven by needs.

b. Based on logic, creativity, and intuition d. Guided by standards e. Focused on safety and quality f. Driven by needs

A nurse assesses clients on a medical-surgical unit, all of whom have stage 2 or 3 pressure injuries. Which client would the nurse evaluate further for a wound infection? Select one: a. WBC 9200 mm/L3 (9.2 × 109) b. Boggy feel to granulation tissue c. Increased size after debridement d. Requesting pain medication

b. Boggy feel to granulation tissue

A nurse is caring for a postoperative client on the surgical unit. The client's blood pressure was 142/76 mm Hg 30 minutes ago, and now is 88/50 mm Hg. What action would the nurse take first? a. Document and continue to monitor. b. Call the Rapid Response Team. c. Notify the primary healthcare provider. d. Repeat the blood pressure in 15 min.

b. Call the Rapid Response Team.

A nurse caring for clients with systemic lupus erythematosus (SLE) plans care understanding the most common causes of death for these clients is which of the following? (Select all that apply.) Select one or more: a. Infection b. Cardiovascular impairment c. Vasculitis d. Chronic kidney disease e. Liver failure f. Blood dyscrasias

b. Cardiovascular impairment d. Chronic kidney disease

A nurse is caring for a client with HIV-III who was admitted with HAND. What sign or symptom would be most important for the nurse to report to the primary health care provider? Select one: a. Nausea b. Change in pupil size c. Weeping open lesions d. Cough

b. Change in pupil size

A client with an infection has a fever. What actions by the nurse help increase the client's comfort? (Select all that apply.) Select one or more: a. Administer antipyretics around the clock. b. Change the client's gown and linens when damp. c. Offer cool fluids to the client frequently. d. Place ice bags in the armpits and groin. e. Provide a fan to help cool the client. f. Sponging the client with tepid water.

b. Change the client's gown and linens when damp. c. Offer cool fluids to the client frequently. f. Sponging the client with tepid water.

After teaching a client who has a stage 2 pressure injury, a nurse assesses the client's understanding. Which dietary choice by the client indicates a good understanding of the teaching? Select one: a. Green salad, a banana, whole wheat dinner roll, coffee b. Chicken breast, broccoli, baked potato, ice water c. Vegetable lasagna and green salad, iced tea d. Hamburger, fruit cup, cookie, diet pop

b. Chicken breast, broccoli, baked potato, ice water

A client who had a fractured ankle open reduction internal fixation (ORIF) 4 weeks ago reports burning pain and tingling in the affected foot. For which potential complication would the nurse anticipate? Select one: a. Delayed bone healing b. Complex regional pain syndrome c. Peripheral neuropathy d. Compartment syndrome

b. Complex regional pain syndrome

A client with HIV-III is admitted to the hospital with Toxoplasma gondii infection. Which action by the nurse is most appropriate? Select one: a. Initiate Contact Precautions. b. Conduct frequent neurologic assessments. c. Conduct frequent respiratory assessments. d. Initiate Protective Precautions.

b. Conduct frequent neurologic assessments.

A nurse is caring for a client who has a serum calcium level of 14 mg/dL (3.5 mmol/L). Which primary health care provider order does the nurse implement first? Select one: a. Encourage oral fluid intake. b. Connect the client to a cardiac monitor. c. Assess urinary output. d. Administer oral calcitonin.

b. Connect the client to a cardiac monitor.

A nurse evaluates the following data in a client's chart: A 78-year-old male with a past medical history of atrial fibrillation is admitted with a chronic leg wound Warfarin sodium (Coumadin) Sotalol (Betapace) Negative-pressure wound therapy (NPWT) to leg wound Based on this information, which action would the nurse take first? Select one: a. Assess the client's vital signs and initiate continuous telemetry monitoring. b. Contact the primary health care provider to discuss the treatment c. Consult the wound care nurse to apply the VAC device. d. Obtain a prescription for a low-fat, high-protein diet with vitamin supplements.

b. Contact the primary health care provider to discuss the treatment

The nurse is caring for clients on the medical-surgical unit. What action by the nurse will help prevent a client from having a type II hypersensitivity reaction? Select one: a. Administering steroids for a positive TB test b. Correctly identifying the client prior to a blood transfusion c. Keeping the client free of the offending agent d. Providing a latex-free environment for the client

b. Correctly identifying the client prior to a blood transfusion

The nurse is caring for a client with peritonitis. What assessment findings would the nurse expect? (Select all that apply.) a. Distended rigid abdomen b. Decreased urinary output c. Fever d. Abdominal pain e. Bradycardia f. Nausea and vomiting

b. Decreased urinary output c. Fever d. Abdominal pain e. Bradycardia f. Nausea and vomiting

A nurse asks a more seasoned colleague to explain best practices when communicating with a person from the lesbian, gay, bisexual, transgender, and questioning/queer (LGBTQ) community. What answer by the faculty is most accurate? a. Avoid embarrassing the client by asking questions. b. Don't make assumptions about his or her health needs. c. Most LGBTQ people do not want to share information. d. No differences exist in communicating with this population.

b. Don't make assumptions about his or her health needs.

The nurse learning about cellular regulation understands that which process occurs during the S phase of the cell cycle? Select one: a. Actual division (mitosis) b. Doubling of DNA c. Growing extra membrane d. No reproductive activity

b. Doubling of DNA

A nurse is orienting a new client and family to the medical-surgical unit. What information does the nurse provide to best help the client promote his or her own safety? a. Have the patient monitor hand hygiene in caregivers. b. Encourage the client and family to be active partners. c. Offer the family the opportunity to stay with the patient. d. Tell the client to always were his or her armband.

b. Encourage the client and family to be active partners.

A client has received several doses of midazolam. The nurse assesses the client to be difficult to arouse with respirations of 6 breaths/min. What actions by the nurse are most important? (Select all that apply.) a. Obtain one dose of flumazenil. b. Ensure suction is working c. Administer oxygen per protocol. d. Monitor client every 10 to 15 minutes for the next 2 hours. e. Obtain naloxone, 0.04 mg for IV push. f. Transfer the client to intensive care.

b. Ensure suction is working c. Administer oxygen per protocol. d. Monitor client every 10 to 15 minutes for the next 2 hours.

A client is having a catheter placed to deliver chemotherapy beads into a liver tumor via the femoral artery. What action by the nurse is most important? Select one: a. Assessing the client's abdomen beforehand b. Ensuring that informed consent is on the chart c. Marking the client's bilateral pedal pulses d. Reviewing client teaching done previously

b. Ensuring that informed consent is on the chart

A nurse is learning about the types of different cells involved in the inflammatory response. Which principles does the nurse learn? (Select all that apply.) Select one or more: a. Basophils are only involved in the general inflammatory process. b. Eosinophils increase during allergic reactions and parasitic invasion. c. Macrophages can participate in many episodes of phagocytosis. d. Monocytes turn into macrophages after they enter body tissues. e. Neutrophils can only take part in one episode of phagocytosis.

b. Eosinophils increase during allergic reactions and parasitic invasion. c. Macrophages can participate in many episodes of phagocytosis. d. Monocytes turn into macrophages after they enter body tissues. e. Neutrophils can only take part in one episode of phagocytosis.

A nurse is caring for a client with an electrical burn. The client has entrance wounds on the hands and exit wounds on the feet. What information is most important to include when planning care? Select one: a. The client may have memory and cognitive issues postburn. b. Everything between the entry and exit wounds can be damaged. c. The respiratory system requires close monitoring for signs of swelling. d. Electrical burns increase the risk of developing future cancers.

b. Everything between the entry and exit wounds can be damaged.

The nurse assesses a client with long-term rheumatoid arthritis (RA) for late signs and symptoms. Which assessment findings will the nurse document as late signs and symptoms of RA? (Select all that apply.) Select one or more: a. Anorexia b. Felty syndrome c. Joint deformity d. Low-grade fever e. Weight loss

b. Felty syndrome c. Joint deformity e. Weight loss

A client with cancer has anorexia and mucositis, and is losing weight. The client's family members continually bring favorite foods to the client and are distressed when the client won't eat them. What action by the nurse is best? Select one: a. Explain the pathophysiologic reasons behind the client not eating. b. Help the family show other ways to demonstrate love and caring. c. Suggest foods and liquids the client might be willing to try to eat. d. Tell the family the client isn't able to eat now no matter what they bring.

b. Help the family show other ways to demonstrate love and caring.

A nurse wishes to work in a community-based practice setting. Which areas would this nurse explore for employment? (Select all that apply.) a. Small community hospital b. Home health care c. Hospice facility d. "Minute clinic" e. Telehealth f. Mobile mammography unit.

b. Home health care c. Hospice facility d. "Minute clinic" e. Telehealth f. Mobile mammography unit.

A hospitalized client has a history of depression for which sertraline is prescribed. The client also has a morphine allergy and a history of alcoholism. After surgery, several opioid analgesics are prescribed. Which one would the nurse choose? a. Meperidine b. Hydromorphone c. Tramadol d. Hydrocodone and acetaminophen

b. Hydromorphone

A nurse is caring for clients with electrolyte imbalances on a medical-surgical unit. Which common causes are correctly paired with the corresponding electrolyte imbalance? (Select all that apply.) Select one or more: a. Hypomagnesemia—kidney failure b. Hyperkalemia—salt substitutes c. Hyponatremia—heart failure d. Hypernatremia—hyperaldosteronism e. Hypocalcemia—diarrhea f. Hypokalemia—loop diuretics

b. Hyperkalemia—salt substitutes c. Hyponatremia—heart failure d. Hypernatremia—hyperaldosteronism e. Hypocalcemia—diarrhea f. Hypokalemia—loop diuretics

A client with rheumatoid arthritis (RA) has an acutely swollen, red, and painful joint. What nonpharmacologic intervention does the nurse recommend? Select one: a. Heating pad b. Ice packs c. Splint d. Paraffin dip

b. Ice packs

A new perioperative nurse is receiving orientation to the surgical area and learns about the Surgical Care Improvement Project (SCIP) goals. What major areas do these measures focus on? (Select all that apply.) a. Malignant hyperthermia testing b. Infection prevention c. Correct hair removal d. Hemorrhage prevention e. Thromboembolism prevention f. Stroke recognition

b. Infection prevention c. Correct hair removal e. Thromboembolism prevention

A client has a platelet count of 9800/mm3 (9800 × 109/L). What action by the nurse is most appropriate? Select one: a. Assess the client for calf pain, warmth, and redness. b. Instruct the client to call for help to get out of bed. c. Obtain cultures as per the facility's standing policy. d. Place the client on protective Isolation Precautions.

b. Instruct the client to call for help to get out of bed.

A nurse is participating in primary prevention efforts directed against cancer. In which activities is this nurse most likely to engage? (Select all that apply.) Select one or more: a. Demonstrating breast self-examination methods to women b. Instructing people on the use of chemoprevention c. Providing vaccinations against certain cancers d. Screening teenage girls for cervical cancer e. Teaching teens the dangers of tanning booths f. Educating adults about healthy eating habits

b. Instructing people on the use of chemoprevention c. Providing vaccinations against certain cancers e. Teaching teens the dangers of tanning booths f. Educating adults about healthy eating habits

A nurse performs a skin screening for a client who has numerous skin lesions. Which lesion does the nurse evaluate first? Select one: a. Beige freckles on the backs of both hands. b. Irregular mole with multiple colors on the leg. c. Large cluster of pustules in the right axilla. d. Thick, reddened papules covered by white scales

b. Irregular mole with multiple colors on the leg.

A nurse obtains health histories when admitting clients to a medical-surgical unit. With which client would the nurse discuss predisposition genetic testing? a. Pregnant woman whose father has sickle cell disease. b. Middle-age woman whose mother died at age 48 of breast cancer. c. Middle-age man of Eastern European Jewish ancestry. d. Young man who has all the symptoms of rheumatoid arthritis.

b. Middle-age woman whose mother died at age 48 of breast cancer.

A nurse plans care for a client who is at risk for infection. Which interventions will the nurse implement to prevent infection? (Select all that apply.) Select one or more: a. Administer prophylactic antibiotics. b. Monitor white blood cell count and differential. c. Screen all visitors for infections. d. Implement Transmission-Based Precautions. e. Promote sufficient nutritional intake.

b. Monitor white blood cell count and differential. c. Screen all visitors for infections. e. Promote sufficient nutritional intake.

A nurse receives report from the laboratory on a client who was admitted for fever. The laboratory technician states that the client has "a shift to the left" on the white blood cell count. What action by the nurse is most important? Select one: a. Document findings and continue monitoring. b. Notify the primary health care provider and request antibiotics. c. Place the client in protective isolation. d. Tell the client this signifies inflammation.

b. Notify the primary health care provider and request antibiotics.

A nurse cares for a client with a recently fractured tibia. Which assessment would alert the nurse to take immediate action? Select one: a. Pain of 4 on a scale of 0-10 b. Numbness in the extremity c. Swollen extremity at the injury site d. Feeling cold while lying in bed

b. Numbness in the extremity

A client asks the nurse about having a total knee arthroplasty to relieve joint pain. Which factor would place the client at the highest risk for impaired postoperative healing? Select one: a. Controlled hypertension b. Obesity c. Osteoarthritis d. Mild osteopenia

b. Obesity

A nurse learns that the fastest growing subset of the older population is which group? a. Young old b. Old old c. Elite old d. Middle old

b. Old old

A postoperative client has an epidural infusion of morphine and bupivacaine. Which actions does the nurse delegate to the assistive personnel (AP)? (Select all that apply.) a. Ask the client to point out any areas of numbness or tingling. b. Perform a bladder scan if the client is unable to void after 4 hours. c. Take and record the client's vital signs per agency protocol. d. Remind the client to use the incentive spirometer every hour. e. Determine how many people are needed to ambulate the client.

b. Perform a bladder scan if the client is unable to void after 4 hours. c. Take and record the client's vital signs per agency protocol. d. Remind the client to use the incentive spirometer every hour.

A nurse caring for an older client on a medical-surgical unit notices the client reports frequent constipation and only wants to eat softer foods such as rice, bread, and puddings. What assessment would the nurse perform first? a. Auscultate bowel sounds. b. Perform an oral assessment. c. Weigh the client. d. Check skin turgor.

b. Perform an oral assessment.

A nurse is caring for a client with a peripheral vascular access device who is experiencing pain, redness, and swelling at the site. After removing the device, what action will the nurse take to relieve pain? Select one: a. Administer topical lidocaine to the site. b. Place warm compresses on the site. c. Administer prescribed oral pain medication. d. Massage the site with scented oils.

b. Place warm compresses on the site.

A client has a bone density score of −2.8. What intervention would the nurse anticipate based on this assessment? Select one: a. Asking the client to complete a food diary b. Planning to teach about bisphosphonates c. Scheduling another scan in 2 years d. Scheduling another scan in 6 months

b. Planning to teach about bisphosphonates

A nurse is caring for several clients at risk for fluid imbalances. Which laboratory results are paired with the correct potential imbalance?(Select all that apply.) Select one or more: a. Sodium: 160 mEq/L (mmol/L): Overhydration b. Potassium: 5.4 mEq/L (mmol/L): Dehydration c. Osmolarity: 250 mOsm/L: Overhydration d. Hematocrit: 68%: Dehydration e. BUN: 39 mg/dL: Overhydration f. Magnesium: 0.8 mg/dL: Dehydration

b. Potassium: 5.4 mEq/L (mmol/L): Dehydration c. Osmolarity: 250 mOsm/L: Overhydration d. Hematocrit: 68%: Dehydration f. Magnesium: 0.8 mg/dL: Dehydration

A client has a leg wound that is in Stage II of the inflammatory response. For what sign or symptom does the nurse assess? Select one: a. Noticeable rubor b. Purulent drainage c. Swelling and pain d. Warmth at the site

b. Purulent drainage

A nurse is caring for a client whose Braden Scale score is 9. What intervention demonstrates a lack of evidence-based knowledge? Select one: a. Requests a referral to a registered dietitian nutritionist. b. Raises the head of the bed no more than 45 degrees. c. Performs perineal cleansing every 2 hours. d. Assesses the client's entire skin surface daily.

b. Raises the head of the bed no more than 45 degrees.

A nurse evaluates the following data in a client's chart: A 66-year-old male with a health history of a cerebral vascular accident and left-side paralysis White blood cell count: 8000/mm3(8 × 109/L)Prealbumin: 15.2 mg/dL (152 mg/L)Albumin: 4.2 mg/dL (42 mg/L)Lymphocyte count: 2000/mm3 (2 × 109/L) Sacral ulcer: 4 × 2 × 1.5 cm Based on this information, which action would the nurse take? Select one: a. Perform a neuromuscular assessment. b. Request a dietary consult. c. Initiate Contact Precautions. d. Assess the client's vital signs.

b. Request a dietary consult.

A client with long-standing heart failure being treated for cancer has received a dose of ondansetron for nausea. What action by the nurse is most important? Select one: a. Assess the client for a headache or dizziness. b. Request a prescription for cardiac monitoring c. Instruct the client to change positions slowly. d. Weigh the client daily before eating.

b. Request a prescription for cardiac monitoring

An older adult recently retired and reports "being depressed and lonely." What information would the nurse assess as a priority? a. History of previous depression b. Role of work in the adult's life c. Previous stressful events d. Usual leisure time activities

b. Role of work in the adult's life

A nurse assesses a client who is prescribed a medication that inhibits aldosterone secretion and release. For which potential complications will the nurse assess? (Select all that apply.) Select one or more: a. Urine output of 25 mL/hr b. Serum potassium level of 5.4 mEq/L (5.4 mmol/L) c. Urine specific gravity of 1.02 g/mL d. Serum sodium level of 128 mEq/L (128 mmol/L) e. Blood osmolality of 250 mOsm/kg (250 mmol/kg)

b. Serum potassium level of 5.4 mEq/L (5.4 mmol/L) e. Blood osmolality of 250 mOsm/kg (250 mmol/kg)

A nurse reviews the electronic health record of a client who has Crohn disease and a draining fistula. Which documentation would alert the nurse to urgently contact the primary health care provider for additional prescriptions? a. Client ate 20% of breakfast meal b. Serum potassium of 2.6 mEq/L (2.6 mmol/L) c. White blood cell count of 8200/mm3 (8.2 × 109/L) d. Client's weight decreased by 3 lb (1.4 kg)

b. Serum potassium of 2.6 mEq/L (2.6 mmol/L)

A nurse assesses a client who has psoriasis. Which action would the nurse take first? Select one: a. Don gloves and an isolation gown. b. Shake the client's hand and introduce self. c. Assess for signs and symptoms of infections. d. Ask the client if she might be pregnant.

b. Shake the client's hand and introduce self.

A client has been placed on Contact Precautions. The client's family is very afraid to visit for fear of being "contaminated" by the client. What action by the nurse is best? Select one: a. Explain to them that these precautions are mandated by law. b. Show the family how to avoid spreading the disease. c. Reassure the family that they will not get the infection. d. Tell the family it is important that they visit the client.

b. Show the family how to avoid spreading the disease.

Nurses at a conference learn the process by which pain is perceived by the client. Which processes are included in the discussion? (Select all that apply.) a. Induction b. Transduction c. Transition d. Transmission e. Modulation

b. Transduction c. Transition d. Transmission e. Modulation

The nurse assesses a client who is admitted with a pelvic fracture. Which assessments would the nurse monitor to prevent or detect a complication of this injury? (Select all that apply.) Select one or more: a. Temperature b. Urinary output c. Blood pressure d. Pupil reaction e. Skin color

b. Urinary output c. Blood pressure e. Skin color

A nurse working in an Acute Care of the Elderly unit learns that frailty in the older population includes which components? (Select all that apply.) a. Weight gain b. Weakness c. Exhaustion d. Frequent illness e. Dementia f. Slowed physical activity

b. Weakness c. Exhaustion f. Slowed physical activity

A nurse teaches a client who is prescribed a central vascular access device and is transferring to a skilled facility for long-term treatment. Which statement will the nurse include in this client's teaching? Select one: a. "You will need to wear a sling on your arm while the device is in place." b. "There is no risk of infection because sterile technique will be used during insertion." c. "Ask all providers to vigorously clean the connections prior to accessing the device." d. "You will not be able to take a bath with this vascular access device."

c. "Ask all providers to vigorously clean the connections prior to accessing the device."

A nurse teaches a client who is at risk for hyponatremia. Which statement does the nurse include in this client's teaching? Select one: a. "Have you spouse watch you for irritability and anxiety." b. "Notify the clinic if you notice muscle twitching." c. "Call your primary health care provider for diarrhea." d. "Bake or grill your meat rather than frying it."

c. "Call your primary health care provider for diarrhea."

A nurse asks the charge nurse to explain the difference between critical thinking and clinical judgment. What statement by the charge nurse is best? a. "Clinical judgment is often clouded by erroneous hypotheses." b. "Critical thinking is the highest level of nursing judgment." c. "Clinical judgment is the observable outcome of critical thinking." d. "Critical thinking requires synthesizing interactions within a situation."

c. "Clinical judgment is the observable outcome of critical thinking."

After teaching a client who is recovering from a colon resection to treat early-stage colorectal cancer (CRC), the nurse assesses the client's understanding. Which statements by the client indicate understanding of the teaching? (Select all that apply.) a. "I will use warm water and a soft washcloth to clean around the stoma." b. "I must change the ostomy appliance daily and as needed." c. "I will make sure that I make lifestyle changes to prevent constipation." d. "I will be sure to have the recommended colonoscopies." e. "I might start bicycling and swimming again once my incision has healed."

c. "I will make sure that I make lifestyle changes to prevent constipation." d. "I will be sure to have the recommended colonoscopies." e. "I might start bicycling and swimming again once my incision has healed."

The nurse is teaching a client who is planning to have a total hip arthroplasty. What statement by the client indicates a need for further teaching? Select one: a. "I will get an IV antibiotic right before surgery to prevent infection." b. "I may request a regional nerve block as part of the surgical anesthesia." c. "I will receive IV heparin before surgery to decrease the risk of clots." d. "I will receive tranexamic acid to help reduce blood loss during surgery."

c. "I will receive IV heparin before surgery to decrease the risk of clots."

A client is prescribed celecoxib for joint pain. What statement by the client indicates a need for further teaching? Select one: a. "I'll report any signs of bleeding or bruising to my primary health care provider." b. "I'll take this drug only as prescribed by my primary health care provider." c. "I'll be sure to take this drug three times a day only on an empty stomach." d. "I'll monitor the amount of urine that I excrete every day and report any changes."

c. "I'll be sure to take this drug three times a day only on an empty stomach."

A nurse has taught a client about dietary changes that can reduce the chances of developing cancer. What statement by the client indicates the nurse needs to provide additional teaching? Select one: a. "Foods high in vitamin A and vitamin C are important." b. "I'll have to cut down on the amount of bacon I eat." c. "I'm so glad I don't have to give up my juicy steaks." d. "Vegetables, fruit, and high-fiber grains are important."

c. "I'm so glad I don't have to give up my juicy steaks."

A client is typed and crossmatched for a unit of blood. Which statement by the nurse indicates a need for further genetic education? a. "Each blood type allele is inherited from the mother or the father." b. "If the client has dominant and recessive alleles, the dominant will be expressed." c. "If the patient's blood type is AB, then the client is homozygous for that trait." d. "Blood type is formed from three gene alleles: A, B, and O."

c. "If the patient's blood type is AB, then the client is homozygous for that trait."

A nurse is confused on why systems thinking is important since working on the unit involves caring for a few specific clients. What explanation by the nurse manager is best? a. "It's a good way to conduct root-cause analysis." b. "You may enter management 1 day and need to know this." c. "It is important for quality improvement and safety." d. "Systems thinking helps you see the bigger picture."

c. "It is important for quality improvement and safety."

A client is receiving rituximab and asks how it works. What response by the nurse is best? Select one: a. "It causes rapid lysis of the cancer cell membranes." b. "It destroys the enzymes needed to create cancer cells." c. "It prevents the start of cell division in the cancer cells." d. "It sensitizes certain cancer cells to chemotherapy."

c. "It prevents the start of cell division in the cancer cells."

A client has a left knee arthrocentesis to remove excess joint fluid. What postprocedure health teaching will the nurse include? Select one: a. "Take your opioid medication as prescribed by the primary health care provider." b. "Do not bear weight on your left leg for at least a week after you get home." c. "Monitor the site for bleeding or clear fluid leakage when you are home." d. "Tell your employer that you can't come back to work for 2 to 3 weeks."

c. "Monitor the site for bleeding or clear fluid leakage when you are home."

The nurse teaches assistive personnel (AP) about care of an older adult diagnosed with osteoporosis. What teaching would the nurse include? Select one: a. "Teach the client to eat high-calcium foods in the diet." b. "Assist the client with activities of daily living." c. "Osteoporosis places the client is at risk for fractures." d. "The client should stay in bed to prevent falling."

c. "Osteoporosis places the client is at risk for fractures."

The primary health care provider prescribes methotrexate (MTX) for a client with a new diagnosis of rheumatoid arthritis. The nurse provides health teaching about the drug. What statement by the nurse is appropriate to include about methotrexate? Select one: a. "It will take at least 1 to 2 weeks for the drug to help relieve your symptoms." b. "The drug is very expensive but there are pharmacy plans to help pay for it." c. "The drug can increase your risk for infection, so you should avoid crowds." d. "It's OK for you to drink about 2 to 3 glasses of wine each week while taking the drug."

c. "The drug can increase your risk for infection, so you should avoid crowds."

A nurse is calling the on-call health care provider about a client who had a hysterectomy 2 days ago and has pain that is unrelieved by the prescribed opioid pain medication. Which statement comprises the background portion of the SBAR format for communication? a. "This client had a vaginal hysterectomy 2 days ago." b. "Dr. Smith doesn't like nonsteroidal anti-inflammatory meds." c. "This client has allergies to morphine and codeine." d. "I would like you to order a different pain medication."

c. "This client has allergies to morphine and codeine."

A client with HIV-III asks the nurse why gabapentin is part of the drug regimen when the client does not have a history of seizures. What response by the nurse is best? Select one: a. "Gabapentin can be used as an antidepressant too." b. "I have no idea why you would be taking this drug." c. "This drug helps treat the pain from nerve irritation." d. "You are at risk for seizures due to fungal infections."

c. "This drug helps treat the pain from nerve irritation."

A client has long-term rheumatoid arthritis that especially affects the hands. The client wants to finish quilting a baby blanket before the birth of her grandchild. What response by the nurse is appropriate? Select one: a. "Let's ask your provider about increasing your pain pills." b. "Hold ice bags against your hands before quilting." c. "Try a paraffin wax dip 20 minutes before you quilt." d. "You need to stop quilting before it destroys your fingers."

c. "Try a paraffin wax dip 20 minutes before you quilt."

A nurse assesses a young female client who is prescribed tazarotene. Which question should the nurse ask prior to starting this therapy? Select one: a. "Do you spend a great deal of time in the sun?" b. "Have you or any family members ever had skin cancer?" c. "Which method of contraception are you using?" d. "Do you drink alcoholic beverages?"

c. "Which method of contraception are you using?"

A client is preparing to have a fecal occult blood test (FOBT). What health teaching would the nurse include prior to the test? a. "This test can determine your genetic risk for developing colorectal cancer." b. "This test will determine whether you have colorectal cancer." c. "You need to avoid red meat and NSAIDs for 48 hours before the test." d. "You don't need to have this test because you can have a virtual colonoscopy."

c. "You need to avoid red meat and NSAIDs for 48 hours before the test."

A nurse cares for a client who has a family history of colorectal cancer. The client states, "My father and my brother had colon cancer. What is the chance that I will get cancer?" How would the nurse respond? a. "If you eat a low-fat and low-fiber diet, your chances decrease significantly." b. "Preemptive surgery and chemotherapy will remove cancer cells and prevent cancer." c. "You should have a colonoscopy more frequently to identify abnormal polyps early." d. "You are safe. This is an autosomal dominant disorder that skips generations."

c. "You should have a colonoscopy more frequently to identify abnormal polyps early."

A client had a bunionectomy with osteotomy. The client asks why healing may take up to 3 months. What explanation by the nurse is best? Select one: a. "The bones in your feet are hard to operate on." b. "The surrounding bones and tissue are damaged." c. "Your feet have less blood flow, so healing is slower." d. "Your feet bear weight so they never really heal."

c. "Your feet have less blood flow, so healing is slower."

A nurse assesses clients on a medical-surgical unit. Which client is at greatest risk for pressure injury development? Select one: a. A 44 year old prescribed IV antibiotics for pneumonia b. A 26 year old who is bedridden with a fractured leg c. A 65 year old with hemiparesis and incontinence d. A 78 year old requiring assistance to ambulate with a walker

c. A 65 year old with hemiparesis and incontinence

he nurse understands which information regarding patient-centered care? a. A project addressing challenges in implementing patient-centered care. b. The ability to use best evidence and practice when making care-related decisions. c. A competency recognizing the client as the source of control of his or her care. d. Purposeful, informed, and outcome-focused care of clients or families.

c. A competency recognizing the client as the source of control of his or her care.

A nurse cares for a client who has a specific mutation in the a1AT (alpha1-antitrypsin) gene. What action would the nurse take? a. Teach the client to perform monthly breast self-examinations and schedule an annual mammogram. b. Support the client when sharing test results and encourages family members to be screened for cancer. c. Advise the client to limit exposure to secondhand smoke and other respiratory irritants. d. Obtain a complete health history to identify other genetic problems associated with this gene mutation.

c. Advise the client to limit exposure to secondhand smoke and other respiratory irritants.

The nurse is caring for a client who has a postoperative paralytic ileus following abdominal surgery. What drug is appropriate to manage this nonmechanical bowel obstruction? a. Amlodipine b. Alosetron c. Alvimopan d. Amitiptyline

c. Alvimopan

A nurse cares for a client who is scheduled for genetic testing. What actions would the nurse include in the patient's pretesting assessment? (Select all that apply.) a. Discuss prevention, early detection, and treatment options. b. Assess the client's perception of the test results. c. Assess the client's understanding of the genetic test. d. Obtain physical assessment data relevant to the at-risk disorder. e. Discuss client rights and obligations regarding disclosure of information.

c. Assess the client's understanding of the genetic test. d. Obtain physical assessment data relevant to the at-risk disorder. e. Discuss client rights and obligations regarding disclosure of information.

A nurse cares for a client who recently completed genetic testing that revealed that she has a BRCA1 gene mutation. What actions would the nurse take next? (Select all that apply.) a. Recommend the client complete weekly breast self-examinations. b. Encourage support by sharing the results with family members. c. Assist the client to make a plan for prevention and risk reduction. d. Disclose the information to the medical insurance company. e. Assess the patient's response to the test results. f. Discuss potential risks for other members of her family.

c. Assist the client to make a plan for prevention and risk reduction. e. Assess the patient's response to the test results. f. Discuss potential risks for other members of her family.

The perioperative nurse manager and the postoperative unit manager are concerned about the increasing number of surgical infections in their hospital. What action by the managers is best? a. Encourage staff on both units to provide peer pressure to adhere to hand hygiene policy. b. Hold educational meetings with the nursing and surgical staff on infection prevention. c. Audit charts to see if the Surgical Care Improvement Project (SCIP) outcomes were met. d. Monitor staff on both units for consistent adherence to established hand hygiene practices.

c. Audit charts to see if the Surgical Care Improvement Project (SCIP) outcomes were met.

What factor best predicts a nurse's willingness to employ critical thinking? a. Skills b. Knowledge c. Caring d. Presence

c. Caring

A nurse assesses clients in an osteoporosis clinic. Which client would the nurse assess first? Select one: a. Client taking calcium with vitamin D who reports flank pain 2 weeks ago. b. Client taking ibandronate who cannot remember when the last dose was. c. Client taking raloxifene who reports unilateral calf swelling. d. Client taking risedronate who reports occasional dyspepsia.

c. Client taking raloxifene who reports unilateral calf swelling.

A nurse working in a medical home would do which of the following as part of the job? a. Advocate with insurance companies. b. Provide out-of-network specialty referrals. c. Coordinate interprofessional care. d. Hold monthly team meetings.

c. Coordinate interprofessional care.

A nurse is caring for clients as a member of the rehabilitation team. Which activities would the nurse complete as part of the nurse's role? (Select all that apply.) a. Assist clients to identify support services and resources for the coordination of services. b. Counsel clients and family members on strategies to cope with disability. c. Coordinate rehabilitation team activities to ensure implementation of the plan of care. d. Support the client's choices by acting as an advocate for the client and family. e. Maintain the function of assistive technology by making needed repairs.

c. Coordinate rehabilitation team activities to ensure implementation of the plan of care. d. Support the client's choices by acting as an advocate for the client and family.

The nurse is teaching a client how to avoid the formation of hemorrhoids. What lifestyle change would the nurse include? a. Quitting smoking b. Avoiding alcohol c. Decreasing fluid intake d. Increasing dietary fiber

c. Decreasing fluid intake

A nurse caring for an older client in the hospital is concerned the client is not competent to give consent for upcoming surgery. What action by the nurse is best? a. Have the client's family sign the consent. b. Do not allow the client to sign the consent. c. Discuss concerns with the health care team. d. Call Adult Protective Services.

c. Discuss concerns with the health care team.

An older adult has diabetic neuropathy and often reports unbearable foot pain. About which medication would the nurse plan to educate the client? a. Morphine sulfate b. Desipramine c. Duloxetine d. Nortriptyline

c. Duloxetine

A nurse working with older adults in the community plans programming to improve morale and emotional health in this population. What activity would best meet this goal? a. Social events such as dances and group dinners b. Financial planning seminar series for older adults c. Exercise program to improve physical function d. Workshop on prevention from becoming an abuse victim

c. Exercise program to improve physical function

A nurse is talking with a co-worker who is moving to a new state and needs to find new employment there. What advice by the nurse is best? a. Ask the hospitals there about standard nurse-client ratios. b. Work in a facility affiliated with a medical or nursing school. c. Find a hospital that has achieved Magnet status. d. Choose the hospital that has the newest technology.

c. Find a hospital that has achieved Magnet status

A client is admitted with superior vena cava syndrome. What action by the nurse is most appropriate? Select one: a. Administer a dose of allopurinol. b. Assess the client's serum potassium level. c. Gently inquire about advance directives. d. Prepare the client for emergency surgery.

c. Gently inquire about advance directives.

A nurse is learning the difference between normal cells and benign tumor cells. What information does this include? Select one: a. Benign tumors grow through invasion of other tissue. b. Benign tumors have lost their cellular regulation from contact inhibition. c. Growing in the wrong place or time is typical of benign tumors. d. The loss of characteristics of the parent cells is called anaplasia.

c. Growing in the wrong place or time is typical of benign tumors.

A nurse assesses a client who has ulcerative colitis and severe diarrhea. Which assessment would the nurse complete first? a. Inspection of oral mucosa b. Recent dietary intake c. Heart rate and rhythm d. Percussion of abdomen

c. Heart rate and rhythm

A nurse is caring for a client recovering from an above-the-knee amputation of the right leg. The client reports pain in the right foot. Which prescribed medication would the nurse most likely administer? Select one: a. Intravenous morphine b. Oral acetaminophen c. Intravenous calcitonin d. Oral ibuprofen

c. Intravenous calcitonin

A postoperative client has respiratory depression after receiving morphine for pain. Which medication and dose does the nurse prepare to administer? a. Naloxone 4 to 20 mg b. Flumazenil 2 to 10 mg c. Naloxone 0.4 to 2 mg d. Flumazenil 0.2 to 1 mg

c. Naloxone 0.4 to 2 mg

The nurse understands that which type of immunity is the longest acting? Select one: a. Artificial active b. Inflammatory c. Natural active d. Natural passive

c. Natural active

After a total knee arthroplasty, a client is on the postoperative nursing unit with a continuous femoral nerve blockade. On assessment, the nurse notes the skin of both legs is pale pink, warm, and dry, but the client is unable to dorsiflex or plantarflex the surgical foot. What action would the nurse take next? Select one: a. Document the findings and monitor as prescribed. b. Increase the frequency of monitoring the client. c. Notify the surgeon or anesthesia provider immediately. d. Palpate the client's bladder or perform a bladder scan.

c. Notify the surgeon or anesthesia provider immediately.

A nurse works on the postoperative floor and has four clients who are being discharged tomorrow. Which one has the greatest need for the nurse to consult other members of the health care team for postdischarge care? a. Married young adult who is the primary caregiver for children. b. Middle-age client who is post-knee replacement, and needs physical therapy. c. Older adult who lives alone at home despite some memory loss. d. Young client who lives alone, and has family and friends nearby.

c. Older adult who lives alone at home despite some memory loss.

A nurse is observing as an assistive personnel (AP) performs hygiene and provides comfort measures to a client with an infection. What action by the AP requires intervention by the nurse? Select one: a. Not using gloves while combing the client's hair b. Rinsing the client's commode pan after use c. Ordering an oscillating fan for the client d. Wearing gloves when providing perianal care

c. Ordering an oscillating fan for the client

The nurse assesses a patient who is recovering from an ileostomy placement. Which assessment finding would alert the nurse to immediately contact the primary health care provider? a. Liquid stool b. Blood-tinged output c. Pale and bluish stoma d. Ostomy pouch intact

c. Pale and bluish stoma

An older client with diabetes is admitted with a heavily draining leg wound. The client's white blood cell count is 38,000/mm3 (38 × 109/L) but the client is afebrile. Which nursing action is most appropriate at this time? Select one: a. Administer acetaminophen as needed. b. Educate the client on amputation. c. Place the client on Contact Precautions. d. Refer the client to the wound care nurse.

c. Place the client on Contact Precautions.

A client with known HIV-II is admitted to the hospital with fever, night sweats, and severe cough. Laboratory results include a CD4+ cell count of 180/mm3 and a negative tuberculosis (TB) skin test 4 days ago. What action would the nurse take first? Select one: a. Initiate Droplet Precautions for the client. b. Notify the primary health care provider about the CD4+ results. c. Place the client under Airborne Precautions. d. Use Standard Precautions to provide care.

c. Place the client under Airborne Precautions.

A nurse is caring for four clients. After the hand-off report, which client would the nurse see first? Select one: a. Client with osteoporosis and a white blood cell count of 27,000/mm3 (27 × 109/L) b. Client with osteoporosis and a bone fracture who requests pain medication c. Post-microvascular bone transfer client whose distal leg is cool and pale d. Client with suspected bone tumor who just returned from having a spinal CT

c. Post-microvascular bone transfer client whose distal leg is cool and pale

A nurse cares for a client who has a serum potassium of 6.5 mEq/L (6.5 mmol/L) and is exhibiting cardiovascular changes. Which intervention will the nurse implement first? Select one: a. Prepare to administer patiromer by mouth. b. Provide a heart-healthy, low-potassium diet. c. Prepare to administer dextrose 20% and 10 units of regular insulin IV push. d. Prepare the client for hemodialysis treatment.

c. Prepare to administer dextrose 20% and 10 units of regular insulin IV push.

A nurse cares for a pregnant client who has a family history of sickle cell disease. The client is unsure if she wants to participate in genetic testing. What action would the nurse take? a. Empathize with the client and share a personal story about a hereditary disorder. b. Advocate for the client and her baby by encouraging genetic testing. c. Provide information about the risks and benefits of genetic testing. d. Teach the client that early detection can minimize transmission to the fetus.

c. Provide information about the risks and benefits of genetic testing.

The nurse learns that the most important function of inflammation and immunity is which purpose? Select one: a. Destroying bacteria before damage occurs b. Preventing any entry of foreign material c. Providing maximum protection against infection d. Regulating the process of self-tolerance

c. Providing maximum protection against infection

A nurse working on a cardiac unit delegated taking vital signs to an experienced assistive personnel (AP). Four hours later, the nurse notes that the client's blood pressure taken by the AP was much higher than previous readings, and the client's mental status has changed. What action by the nurse would most likely have prevented this negative outcome? a. Taking the blood pressure instead of delegating the task. b. Double-checking the AP by taking another blood pressure. c. Providing more appropriate supervision of the AP. d. Determining if the AP knew how to take blood pressure.

c. Providing more appropriate supervision of the AP.

A nurse admits an older adult from a home environment. The client lives with an adult son and daughter-in-law. The client has urine burns on the skin, no dentures, and several pressure injuries. What action by the nurse is most appropriate? a. Call the police department and file a report. b. Notify Adult Protective Services. c. Report the findings as per agency policy. d. Ask the family how these problems occurred.

c. Report the findings as per agency policy.

To demonstrate clinical reasoning skills, what action does the nurse take? a. Collaborating with co-workers to buddy up for lunch breaks. b. Delegating frequent vital signs on a new postoperative patient. c. Requesting the provider order medication for a client with high potassium. d. Documenting a complete history and physical on an admission.

c. Requesting the provider order medication for a client with high potassium.

The nurse is caring for a client with a large bowel obstruction due to fecal impaction. What position would be appropriate for the client while in bed? a. Supine b. Prone c. Semi-Fowler d. Recumbent

c. Semi-Fowler

A client is admitted with a diagnosis of possible strangulated inguinal hernia. For which complication would the nurse monitor? a. Colitis b. Bowel volvulus c. Sepsis d. Paralytic ileus

c. Sepsis

Four clients are receiving tyrosine kinase inhibitors (TKIs). Which of these four clients would the nurse assess first? Select one: a. Dry, itchy, peeling skin b. Serum calcium of 9.2 mg/dL (2.3 mmol/L) c. Serum potassium of 2.8 mEq/L (2.8 mmol/L) d. Weight gain of 0.5 lb (1.1 kg) in 1 day

c. Serum potassium of 2.8 mEq/L (2.8 mmol/L)

A new nurse is preparing to administer IV potassium to a client with hypokalemia. What action indicates the nurse needs to review this procedure? Select one: a. Notifies the pharmacy of the IV potassium order. b. Assesses the client's IV site every hour during infusion. c. Sets the IV pump to deliver 30 mEq of potassium an hour. d. Double-checks the IV bag against the order with the precepting nurse.

c. Sets the IV pump to deliver 30 mEq of potassium an hour.

The nurse is performing an assessment of a client with possible plantar fasciitis in the right foot. What assessment finding would the nurse expect in the right foot? Select one: a. Multiple toe deformities b. Numbness and paresthesias c. Severe pain in the arch of the foot d. Redness and severe swelling

c. Severe pain in the arch of the foot

The nurse assesses a client who has appendicitis. Which assessment finding would the nurse expect? a. Abdominal pain that increases with knee flexion b. Marked peristalsis and hyperactive bowel sounds c. Severe, steady right lower quadrant pain d. Abdominal pain associated with nausea and vomiting

c. Severe, steady right lower quadrant pain

After administering potassium chloride, a nurse evaluates the client's response. Which signs and symptoms indicate that treatment is improving the client's hypokalemia? (Select all that apply.) Select one or more: a. Respiratory rate of 8 breaths/min b. Absent deep tendon reflexes c. Strong productive cough d. Active bowel sounds e. U waves present on the electrocardiogram (ECG)

c. Strong productive cough d. Active bowel sounds

A home health care nurse has conducted a home safety assessment for an older adult. There are five concrete steps leading out from the front door. Which intervention would be most helpful in keeping the older adult safe on the steps? a. Tell the client to use a two-footed gait on the steps. b. Instruct the client to use the garage door instead. c. Teach the client to hold the handrail when using the steps d. Have the client use a walker or cane on the steps.

c. Teach the client to hold the handrail when using the steps

The postanesthesia care unit (PACU) nurse is caring for an older client following a lengthy surgery. The client's pulse is 48 beats/min which is 20 beats/min lower than the preoperative baseline. What assessment does the nurse make next? a. Rate of IV infusion b. Blood pressure c. Temperature d. Level of consciousness

c. Temperature

A nurse is caring for a client who has a flaccid bladder after a spinal cord injury. Which intervention would the nurse implement to assist with bladder dysfunction? a. Stroke the medial aspect of the thigh. b. Insert an indwelling urinary catheter. c. Use the Credé maneuver every 3 hours. d. Apply an external (condom) catheter with a leg bag.

c. Use the Credé maneuver every 3 hours.

A nurse assesses a client recovering from coronary artery bypass graft surgery in an inpatient rehabilitation unit. Which assessment would the nurse complete to evaluate the client's activity tolerance? a. Body image and self-care abilities b. Ability to use assistive or adaptive devices c. Vital signs before, during, and after activity d. Client's electrocardiography readings

c. Vital signs before, during, and after activity

The nurse is caring for a client who is prescribed sulfasalazine. Which question would the nurse ask the client before starting this drug? a. "Can you swallow pills pretty easily?" b. "Do you have insurance to cover this drug?" c. "Are you taking Vitamin C or B? d. "Do you have any allergy to sulfa drugs?"

d. "Do you have any allergy to sulfa drugs?"

A newly graduated nurse in the hospital states that because of being so new, participation in quality improvement (QI) projects is not wise. What response by the precepting nurse is best? a. "All staff nurses are required to participate in quality improvement here." b. "You should ask to be assigned to the research and quality committee." c. "It's easy to identify what indicators would be used to measure quality." d. "Even being new, you can implement activities designed to improve care."

d. "Even being new, you can implement activities designed to improve care."

The nurse receives a hand-off report. One client is described as a drug seeker who is obsessed with even tiny changes in physical condition and is "on the light constantly" asking for more pain medication. When assessing this client's pain, which statement or question by the nurse is most appropriate? a. "Why do you think the medication is not helping your pain?" b. "I wish I could do more; is there anything I can get for you?" c. "You cannot have more pain medication for 3 hours." d. "Help me understand how pain is affecting you right now."

d. "Help me understand how pain is affecting you right now."

A nurse is caring for a client who is recovering from an above-the-knee amputation and reports pain in the limb that was removed. How would the nurse respond? Select one: a. "The pain you are feeling does not actually exist." b. "This type of pain is common and will eventually go away." c. "Would you like to learn how to use imagery to minimize your pain?" d. "How would you describe the pain that you are feeling?"

d. "How would you describe the pain that you are feeling?"

After teaching a client who has diverticulitis, a nurse assesses the client's understanding. Which statement made by the client indicates a need for further teaching? a. "I must try to include at least 25 g of fiber in my diet every day." b. "I should use my legs rather than my back muscles when I lift heavy objects." c. "I'll ride my bike or take a long walk at least three times a week." d. "I will take a laxative nightly at bedtime to avoid becoming constipated."

d. "I will take a laxative nightly at bedtime to avoid becoming constipated."

A nurse manager is preparing an educational session for floor nurses on drug-resistant organisms. Which statement below indicates the need to review this information? Select one: a. "Methicillin-resistant Staphylococcus aureus can be hospital- or community-acquired." b. "Vancomycin-resistant Enterococcus can live on surfaces and be infectious for weeks." c. Carbapenem-resistant Enterobacteriaceae is hard to treat due to enzymes that break down antibiotics." d. "If you leave work wearing your scrubs, go directly home and wash them right away."

d. "If you leave work wearing your scrubs, go directly home and wash them right away."

The nurse has taught a client with cancer ways to prevent infection. What statement by the client indicates that more teaching is needed? Select one: a. "I should take my temperature daily and when I don't feel well." b. "I will discard perishable liquids after sitting out for over an hour." c. "I won't let anyone share any of my personal toiletries." d. "It's alright for me to keep my pets and change the litter box."

d. "It's alright for me to keep my pets and change the litter box."

A client contacts the clinic to report a life-long mole has developed a crust with occasional bleeding. What instruction by the nurse is most appropriate? Select one: a. "Take monthly photographs of it so you can document any changes." b. "Wash daily with warm water and gentle soap to prevent infection." c. "Keep the lesion covered with a bandage and triple antibiotic ointment." d. "Please make an appointment to be seen here as soon as possible."

d. "Please make an appointment to be seen here as soon as possible."

A nurse cares for a client who had a wrist cast applied 3 days ago. The client states, "The cast is loose enough to slide off." How would the nurse respond? Select one: a. "Keep your arm above the level of your heart." b. "As your muscles atrophy, the cast is expected to loosen." c. "I will wrap a bandage around the cast to prevent it from slipping." d. "You need a new cast now that the swelling is decreased."

d. "You need a new cast now that the swelling is decreased."

A nurse is caring for several clients with fractures. Which client would the nurse identify as being at the highest risk for developing deep vein thrombosis? Select one: a. An 18-year-old male athlete with a fractured clavicle b. A 36-year-old female with type 2 diabetes and fractured ribs c. A 55-year-old female prescribed ibuprofen for osteoarthritis d. A 74-year-old male who smokes and has a fractured pelvis

d. A 74-year-old male who smokes and has a fractured pelvis

A nurse is caring for a client with hypocalcemia. Which action by the nurse shows poorunderstanding of this condition? Select one: a. Assesses the client's Chvostek and Trousseau sign. b. Keeps the client's room quiet and dimly lit. c. Moves the client carefully to avoid fracturing bones. d. Administers bisphosphonates as prescribed.

d. Administers bisphosphonates as prescribed.

A nurse is caring for a client who has a nonhealing pressure injury on the right ankle. Which action would the nurse take first? Select one: a. Draw blood for albumin, prealbumin, and total protein. b. Prepare for and assist with obtaining a wound culture. c. Instruct the client to elevate the foot. d. Assess the right leg for pulses, skin color, and temperature.

d. Assess the right leg for pulses, skin color, and temperature.

A client has received an opioid analgesic for pain. The nurse assesses that the client has a Pasero Scale score of 3 and a respiratory rate of 7 shallow breaths/min. The client's oxygen saturation is 87%. Which action would the nurse perform first? a. Notify the Rapid Response Team. b. Give naloxone (Narcan). c. Apply oxygen at 4 L/min. d. Attempt to arouse the client.

d. Attempt to arouse the client.

A postoperative client vomited. After cleaning and comforting the client, which action by the nurse is most important? a. Document the episode. b. Allow the client to rest. c. Encourage the client to eat dry toast. d. Auscultate lung sounds.

d. Auscultate lung sounds.

A client is going to be admitted for a scheduled surgical procedure. Which action does the nurse explain is the most important thing the client can do to protect against errors? a. Keep the provider's telephone number by the phone. b. Make sure that all providers wash hands before entering the room. c. Write down the name of each of the caregivers who come in the room. d. Bring a list of all medications and what they are for.

d. Bring a list of all medications and what they are for.

A home health care nurse is planning an exercise program with an older adult who lives at home independently but whose mobility issues prevent much activity outside the home. Which exercise regimen would be most beneficial to this adult? a. Increasing aerobic capacity b. Improving exercise endurance c. Providing personal training d. Building strength and flexibility

d. Building strength and flexibility

The nurse is caring for a postoperative client who have a regional nerve blockade for a surgical tibial fracture repair this morning. What assessment finding would the nurse expect? Select one: a. Client reports nausea and vomiting. b. Client reports tingling in the surgical leg. c. Client responds well to imagery. d. Client reports little to no pain.

d. Client reports little to no pain.

The nurse is caring for several clients with osteoporosis. For which client would bisphosphonates not be a good option? Select one: a. Client with diabetes who has a serum creatinine of 0.8 mg/dL (61 mcmol/L). b. Client who recently fell and has vertebral compression fractures. c. Hypertensive client who takes calcium channel blockers. d. Client with a spinal cord injury who cannot tolerate sitting up.

d. Client with a spinal cord injury who cannot tolerate sitting up.

A nurse on the medical-surgical unit has received a hand-off report. Which client would the nurse see first? a. Client being discharged later on a complicated analgesia regimen. b. Client who has returned from physical therapy and is resting in the recliner. c. Postoperative client who received oral opioid analgesia 45 minutes ago. d. Client with new-onset abdominal pain, rated as an 8 on a 0-10 scale.

d. Client with new-onset abdominal pain, rated as an 8 on a 0-10 scale.

A nurse working in a medical home would do which of the following as part of the job? a. Advocate with insurance companies. b. Hold monthly team meetings. c. Provide out-of-network specialty referrals. d. Coordinate interprofessional care.

d. Coordinate interprofessional care.

A nurse is evaluating a client who is being treated for dehydration. Which assessment result does the nurse correlate with a therapeutic response to the treatment plan? Select one: a. Increased respiratory rate from 12 to 22 breaths/min b. Decreased skin turgor on the client's posterior hand and forehead c. Increased urine specific gravity from 1.012 to 1.030 g/mL d. Decreased orthostatic changes when standing

d. Decreased orthostatic changes when standing

An inpatient nurse brings an informed consent form to a client for an operation scheduled for tomorrow. The client asks about possible complications from the operation. What response by the nurse is best a. Have the client sign the consent, and then call the primary health care provider. b. Answer the questions and document that teaching was done. c. Remind the client of what teaching the primary health care provider has done. d. Do not have the client sign the consent and call the primary health care provider.

d. Do not have the client sign the consent and call the primary health care provider.

The nurse reviews the laboratory results for a client who has possible appendicitis. Which laboratory test finding would the nurse expect? a. Decreased thrombocyte count b. Decreased potassium level c. Increased sodium level d. Elevated leukocyte count

d. Elevated leukocyte count

A new nurse is working with a preceptor on a medical-surgical unit. The preceptor advises the new nurse that which is the priority when working as a professional nurse? a. Providing patient centered care b. Attending to holistic patient needs c. Not making medication errors d. Ensuring client safety

d. Ensuring client safety

After teaching a client who is prescribed a restricted sodium diet, a nurse assesses the client's understanding. Which food choice for lunch indicates that the client correctlyunderstood the teaching? Select one: a. Slices of smoked ham with potato salad b. Bowl of tomato soup with a grilled cheese sandwich c. Salami and cheese on whole-wheat crackers d. Grilled chicken breast with glazed carrots

d. Grilled chicken breast with glazed carrots

A nurse assesses a client with a pelvic fracture. Which assessment finding would the nurse identify as a complication of this injury? Select one: a. Hypertension b. Diarrhea c. Infection d. Hematuria

d. Hematuria

An older adult client is in the hospital. The client is ambulatory and independent. What intervention by the nurse would be most helpful in preventing falls in this client? a. Order a bedside commode for the client b. Put the client on a toileting schedule. c. Use side rails to keep the client in bed. d. Keep the light on in the bathroom at night.

d. Keep the light on in the bathroom at night.

A nurse wishes to participate in an activity that will influence health outcomes. What action by the nurse best meets this objective? a. Creating a transportation system for health care appointments. b. Organizing a food pantry in an impoverished community. c. Running for election to the county public health board. d. Lobbying with a national organization for health care policy.

d. Lobbying with a national organization for health care policy.

A nurse is assessing a client with glioblastoma. What assessment is most important? Select one: a. Abdominal palpation b. Abdominal percussion c. Lung auscultation d. Neurologic examination

d. Neurologic examination

A nurse is teaching the client with systemic lupus erythematosus about prednisone. What information is the priority? Select one: a. Might make the client feel jittery or nervous. b. Can cause sodium and fluid retention. c. Long-term effects include fat redistribution. d. Never stop prednisone abruptly.

d. Never stop prednisone abruptly.

A client had a surgical procedure with spinal anesthesia. The client's blood pressure was 122/78 mm Hg 30 minutes ago and is now 138/60 and the client reports nausea. What action by the nurse is best? a. Nothing; this is expected. b. Call the Rapid Response Team. c. Increase the IV fluid rate. d. Notify the primary health care provider.

d. Notify the primary health care provider.

An older client who fell at home is admitted to the emergency department and reports pain in her left groin and behind her left knee. What action would the nurse anticipate? Select one: a. Administer IV push morphine. b. Prepare for application of a leg cast. c. Begin oxygen at 6 L/min via mask. d. Obtain a left hip x-ray.

d. Obtain a left hip x-ray.

A client is admitted with possible sepsis. Which action will the nurse perform first? Select one: a. Administer antibiotics. b. Give an antipyretic. c. Place the client in isolation. d. Obtain specified cultures.

d. Obtain specified cultures.

A client is hospitalized with Pneumocystis jiroveci pneumonia. The client reports shortness of breath with activity and extreme fatigue. What intervention is best to promote comfort? Select one: a. Administer sleeping medication. b. Perform most activities for the client. c. Increase the client's oxygen during activity. d. Pace activities, allowing for adequate rest.

d. Pace activities, allowing for adequate rest.

A nurse prepares to insert a peripheral venous catheter in an older adult. What action will the nurse take to protect the client's skin during this procedure? Select one: a. Lower the extremity below the level of the heart. b. Apply warm compresses to the extremity. c. Tap the skin lightly and avoid slapping. d. Place a washcloth between the skin and tourniquet.

d. Place a washcloth between the skin and tourniquet.

A registered nurse is caring for a client who is receiving pain medication via patient-controlled analgesia (PCA). Which action by the nurse indicates the need for further education on pain control with PCA? a. Reinforces client teaching about using the PCA pump. b. Assesses the client's pain level per agency policy. c. Monitors the client's respiratory rate and sedation. d. Presses the button when the client cannot reach it.

d. Presses the button when the client cannot reach it.

A nurse asks the staff development nurse what "apoptosis" means. What response best? Select one: a. Growth by cells enlarging b. Having the normal number of chromosomes c. Inhibition of cell growth d. Programmed cell death

d. Programmed cell death

A new nurse has been assigned a client who is in the hospital to receive iodine-131 treatment. Which action by the nurse is best? Select one: a. Ensure the client is placed in protective isolation. b. Have pregnant visitors stay 6 feet from the client c. No special action is necessary to care for this client. d. Read the policy on handling radioactive excreta.

d. Read the policy on handling radioactive excreta.

A nurse is assessing a female client who is taking hormone therapy for breast cancer. What assessment finding requires the nurse to notify the primary health care provider immediately? Select one: a. Irregular menses b. Edema in the lower extremities c. Ongoing breast tenderness d. Red, warm, swollen calf

d. Red, warm, swollen calf

The nurse working with clients who have autoimmune diseases understands that what component of cell-mediated immunity is the problem? Select one: a. CD4+ cells b. Cytotoxic T-cells c. Natural killer cells d. Regulator T-cells

d. Regulator T-cells

A client is scheduled to have a total hip arthroplasty. What preoperative teaching by the nurse is most important? Select one: a. Teach the need to discontinue all medications for 5 days before surgery. b. Teach the patient about foods high in protein, Vitamin C, and iron. c. Explain to the client the possible need for blood transfusions postoperatively. d. Remind the client to have all dental procedures completed at least 2 weeks prior to surgery.

d. Remind the client to have all dental procedures completed at least 2 weeks prior to surgery.

The nurse is caring for a young client who has been diagnosed with osteopenia. Which risk factor in the client's history most likely contributed to the bone loss? Select one: a. Osteoarthritis b. Hypothyroidism c. Addison disease d. Rheumatoid arthritis

d. Rheumatoid arthritis

After teaching a client with a fractured humerus, the nurse assesses the client's understanding. Which dietary choice demonstrates that the client correctly understands the nutrition needed to assist in healing the fracture? Select one: a. Baked fish with orange juice and a vitamin D supplement b. Bacon, lettuce, and tomato sandwich with a vitamin B supplement c. Vegetable lasagna with a green salad and a vitamin A supplement d. Roast beef with low-fat milk and a vitamin C supplement

d. Roast beef with low-fat milk and a vitamin C supplement

A client who had surgery has extreme postoperative pain that is worsened when trying to participate in physical therapy. Which intervention for pain management does the nurse include in the client's care plan? a. As-needed pain medication after therapy b. Pain medications prior to therapy only c. Patient-controlled analgesia with a basal rate d. Round-the-clock analgesia with PRN analgesics

d. Round-the-clock analgesia with PRN analgesics

A nurse plans care for a client who has an external fixator on the lower leg. Which intervention would the nurse include in the plan of care to decrease the client's risk for infection? Select one: a. Washing the frame of the fixator once a day b. Releasing fixator tension for 30 minutes twice a day c. Avoiding moving the extremity by holding the fixator d. Scheduling for pin care to be provided every shift

d. Scheduling for pin care to be provided every shift

The nurse learning about infection discovers that which factor is the best and mostimportant barrier to infection? Select one: a. Colonization by host bacteria b. Gastrointestinal secretions c. Inflammatory processes d. Skin and mucous membranes

d. Skin and mucous membranes

A nurse assesses a client who has a radial artery catheter. Which assessment will the nurse complete first? Select one: a. Amount of pressure in fluid container b. Date of catheter tubing change c. Type of dressing over the site d. Skin color and capillary refill

d. Skin color and capillary refill

A nurse assesses a client's peripheral IV site, and notices edema and tenderness above the site. What action will the nurse take next? Select one: a. Apply cold compresses to the IV site. b. Elevate the extremity on a pillow. c. Flush the catheter with normal saline. d. Stop the infusion of intravenous fluids.

d. Stop the infusion of intravenous fluids.

A nurse teaches a client who has a reflex (spastic) bladder after a spinal cord injury. Which bladder training technique would the nurse teach? a. Valsalva maneuver b. Self-catheterization c. Frequent toileting d. Stroking the medial aspect of the thigh

d. Stroking the medial aspect of the thigh

A nurse works with clients who have alopecia from chemotherapy. What action by the nurse takes priority? Select one: a. Helping clients adjust to their appearance b. Reassuring clients that this change is temporary c. Referring clients to a reputable wig shop d. Teaching measures to prevent scalp injury

d. Teaching measures to prevent scalp injury

A client is in the oncology clinic for a first visit since being diagnosed with cancer. The nurse reads in the client's chart that the cancer classification is TISN0M0. What does the nurse conclude about this client's cancer? Select one: a. The primary site of the cancer cannot be determined. b. Regional lymph nodes could not be assessed. c. There are multiple lymph nodes involved already. d. There are no distant metastases noted in the report.

d. There are no distant metastases noted in the report.

Which statement about carcinogenesis is accurate? Select one: a. An initiated cell will always become clinical cancer. b. Cancer becomes a health problem once it is 1 cm in size. c. Normal hormones and proteins do not promote cancer growth. d. Tumor cells need to develop their own blood supply.

d. Tumor cells need to develop their own blood supply.

Which statements are true regarding Standard Precautions? (Select all that apply.) Select one or more: a. Always wear a gown when performing hygiene on clients. b. Sneeze into your sleeve or into a tissue that you throw away. c. Remain 3 feet (1 m) away from any client who has an infection. d. Use personal protective equipment as needed for client care. e. Wear gloves when touching clients' excretions or secretions. f. Cohorting clients who have infections caused by the same organism.

d. Use personal protective equipment as needed for client care. e. Wear gloves when touching clients' excretions or secretions.

The nurse assesses a client with rheumatoid arthritis (RA) and Sjögren syndrome. What assessment would be most important for this client? Select one: a. Abdominal assessment b. Oxygen saturation c. Breath sounds d. Visual acuity

d. Visual acuity

A client with cancer is admitted to a short-term rehabilitation facility. The nurse prepares to administer the client's oral chemotherapy medications. What action by the nurse is most appropriate? Select one: a. Crush the medications if the client cannot swallow them. b. Give one medication at a time with a full glass of water. c. No special precautions are needed for these medications. d. Wear personal protective equipment when handling the medications.

d. Wear personal protective equipment when handling the medications.


संबंधित स्टडी सेट्स

Module 00102-15 Exam Introduction to Construction Math

View Set

COMBINED FLASH CARDS FOR RI BOATING EXAM; 1. ri boating license test TR edits; 2. BOAT Licensing Exam Terms use this one separate questions from website

View Set

Examen parcial oral Span 152: Preguntas de prática

View Set

Successions: Inheritance Rights in general

View Set

Issues In Global Business Misterm

View Set

CIVIL LITIGATION - CHAPTER TEST 7-13

View Set

The 8 Planets in our Solar System

View Set

Lecture 6 - Mass and process balancesOb

View Set